Vous êtes sur la page 1sur 68

A TASTE OF MATHEMATICS

AIMETON LES MATHEMATIQUES

Volume / Tome VII

THE MURRAY KLAMKIN PROBLEMS


CANADIAN COLLECTION Part 1.

Edited by

Andy Liu
University of Alberta
and

Bruce Shawyer
Memorial University of Newfoundland

Published by the Canadian Mathematical Society, Ottawa, Ontario


and produced by the CMS ATOM Office, St. Johns, NL
Publie par la Societe mathematique du Canada, Ottawa (Ontario)
et produit par le Bureau ATOM de la SMC, St. Johns, NL

Printed in Canada by / imprime au Canada par


The University of Toronto Press Incorporated
ISBN 0-919558-16-X

All rights reserved.


No part of this publication may be reproduced or
transmitted in any form or by any means, electronic or mechanical, including
photocopying, recording, or any information or retrieval system now known or
to be invented, without permission in writing from the publisher: The Canadian
Mathematical Society, 577 King Edward Avenue, P.O. Box 450, Station A, Ottawa,
Ontario K1N 6N5, Canada, except so far as may be allowed by law.
Tous droits reserves.
Aucune partie de cet ouvrage ne peut etre
reproduite ou utilisee par quelque procede ou quelque facon que ce soit, y compris
les methodes electroniques ou mecaniques, les enregistrements ou les syst`emes de
mise en memoire et dinformation, sans laccord prealable ecrit de lediteur, la
Societe mathematique du Canada, 577, rue King-Edward, C.P. 450, succursale A,
Ottawa (Ontario) K1N 6N5, Canada, sauf dans les limites prescrites par la loi.
c 2005

Canadian Mathematical Society / Societe mathematique du Canada

A TASTE OF MATHEMATICS

AIMETON LES MATHEMATIQUES

Volume / Tome VII

THE MURRAY KLAMKIN PROBLEMS


CANADIAN COLLECTION Part 1.

Edited by

Andy Liu
University of Alberta
and

Bruce Shawyer
Memorial University of Newfoundland

The ATOM series


The booklets in the series, A Taste of Mathematics, are published by
the Canadian Mathematical Society (CMS). They are designed as enrichment
materials for high school students with an interest in and aptitude for
mathematics. Some booklets in the series will also cover the materials useful
for mathematical competitions at national and international levels.

La collection ATOM
Publies par la Societe mathematique du Canada (SMC), les livrets
de la collection Aime-t-on les mathematiques (ATOM) sont destines au
perfectionnement des etudiants du cycle secondaire qui manifestent un interet
et des aptitudes pour les mathematiques. Certains livrets de la collection
ATOM servent egalement de materiel de preparation aux concours de
mathematiques sur lechiquier national et international.

Editorial Board / Conseil de r


edaction
Editor-in-Chief / Redacteur-en-chef
Bruce Shawyer
Memorial University of Newfoundland / Universite Memorial de Terre-Neuve

Associate Editors / Redacteurs associes


Edward J. Barbeau
University of Toronto / Universite de Toronto

Malgorzata Dubiel
Simon Fraser University / Universite Simon Fraser

Joseph Khoury
University of Ottawa / Universite dOttawa

Antony Thompson
Dalhousie University / Universite Dalhousie

Managing Editor / Redacteur-gerant


Graham P. Wright
University of Ottawa / Universite dOttawa

iv

Table of Contents
Preface

1 Quickies

2 Combinatorics and Number Theory

27

3 Functions and Polynomials

39

4 Expressions and Identities

51

QUICKIES
Murray Klamkin was famous for his Quickies, problems that had
quick and neat solutions. We present all the Quickies published in CRUX
MATHEMATICORUM, with some slight editing by Bruce Shawyer.

PROBLEMS AND SOLUTIONS


The problems have been selected by Andy Liu and arranged into sets
according to topic.
The solutions are as published in CRUX MATHEMATICORUM, with some
slight editing by Bruce Shawyer. Solutions from pre-LATEX editions were coded
by students at Memorial University of Newfoundland, funded by the Canadian
Mathematical Society. Special thanks are due to Karelyn Davis, Alyson Ford,
Don Hender, Shawna Gammon, Paul Marshall, Shannon Sullivan and Rebecca
White.
These problems exhibit the special talents of Murray Klamkin. They cover
a very wide range of topics, and show a great deal of insight into what is possible
in there areas. They demonstrate that Murray Klamin was a problem setter par
excellence. We are very greatful to have known him and to have been inspired by
him.
Problem numbers and references are to [year : page number] are as in CRUX
MATHEMATICORUM. When a problem number is followed by a star ?, this
means that the problem was proposed without a solution.

Andy Liu
Department of Mathematics
University of Alberta
Edmonton
Alberta

Bruce Shawyer
Department of Mathematics
Memorial University of Newfoundland
St. Johns
Newfoundland and Labrador

Problems posed by Murray Klamkin.


Unless otherwise stated, these problems were proposed by Murray Klamkin alone.
Quickies
Part 1
154
Combinatorics and Number Theory
Part 1
429, 1456, 2054, 1863, 1027, 969, 1561, 2034, 1752, 1434.
Vectors and Matrices
Part 4
1200, 1721, 1482, 1693, 2005, 398, 1314, 3024, 1207, 1242.
Functions and Polynomials
Part 1
299, 254, 1423, 2014, 1110, 1283.
Expressions and Identities.
Part 1
1304, 287, 1594, 1522, 830, 1996, 1362.
Numerical Approximations
Part 4
1003, 1213, 1371.
Algebraic Inequalities.
Part 2
347, 1642, 2615, 1703, 1734, 1445, 2064, 2095, 2044, 1652,
1742, 1674, 1774, 323, 805, 1394, 2734, 2839, 1662.
Trigonometric Inequalities.
Part 3
1414, 908, 1352, 1712, 1542, 1613, 1503, 2084, 1332, 1801,
1271, 1060, 958, 1962.
Geometric Inequalities.
Part 4
1165, 1473, 1574, 1764, 1296, 506, 1131, 1985, 1945.
The Triangle.
Part 3
1872, 1605, 1385, 2848, 210, 1076, 1532, 2618.
Cevian Lines.
Part 4
2614, 548, 485, 2613, 1621, 1631.
Central Symmetry.
Part 3
1062, 1348, 1513.
Conic Sections.
Part 2
2616, 1975, 1935, 1405, 520.
Solid Geometry.
Part 2
375, 1784, 1553, 1581, 330, 478, 2617, 1261.
Higher Dimensions.
Part 2
2651, 224, 1086, 1465, 2733, 2024, 1793.
Calculus.
Part 4
1178, 1494, 1322, 1147, 273.
Problems dedicated to Murray Klamkin
Part 4
1241, 2619, 2620, 2621
Klamkin Problems of September 2005
Part 4
K01 through K15

Quickies

1.

Determine the extreme values of r1 /h1 + r2 /h2 + r3 /h3 + r4 /h4 where h1 , h2 ,


h3 , h4 are the four altitudes of a given tetrahedron T and r1 , r2 , r3 , r4 are the
corresponding signed perpendicular distances from any point in the space of T to
the faces.

2.

Determine the minimum value of the product


P = (1 + x1 + y1 )(1 + x2 + y2 ) . . . (1 + xn + yn )

where xi , yi 0, and x1 x2 . . . xn = y1 y2 . . . yn = an .

3.

Prove that if F (x, y, z) is a concave function of x, y, z, then {F (x, y, z)}2 is


a convex function of x, y, z.

4.

If a, b, c are sides of a given triangle of perimeter p, determine the maximum


values of

(i) (a b)2 + (b c)2 + (c a)2 ,


(ii) |a b| + |b c| + |c a|,

(iii) |a b||b c| + |b c||c a| + |c a||a b|.

5. If A, B, C are three dihedral angles of a trihedral angle, show that sin A, sin B,
sin C satisfy the triangle inequality.

6.

Are there any integral solutions (x, y, z) of the Diophantine equation


(x y z)3 = 27xyz

other than (a, a, a) or such that xyz = 0?

7.

Does the Diophantine equation


(x y z)(x y + z)(x + y z) = 8xyz

have an infinite number of relatively prime solutions?

8.

It is an easy result using calculus that if a polynomial P (x) is divisible by


its derivative P 0 (x), then P (x) must be of the form a(x r)n . Starting from the
known result that
P 0 (x) X 1
=
P (x)
x ri

where the sum is over all the zeros ri of P (x) counting multiplicities, give a
non-calculus proof of the above result.

9.

Solve the simultaneous equations


x2 (y + z) = 1,

y 2 (z + x) = 8,

z 2 (x + y) = 13.

10.

Determine the area of a triangle of sides a, b, c and semiperimeter s if


(s b)(s c) =

a
,
h

(s c)(s a) =

b
,
k

(s a)(s b) =

c
,
l

where h, k, l are consistent given constants.

11.

Prove that
3(x2 y + y 2 z + z 2 x)(xy 2 + yz 2 + zx2 ) xyz(x + y + z)2

where x, y, z 0.

12.

Determine all integral solutions of the Diophantine equation


(x8 + y 8 + z 8 ) = 2(x16 + y 16 + z 16 ).

13.

Determine all the roots of the quintic equation


31x5 + 165x4 + 310x3 + 330x2 + 155x + 33 = 0.

14.

If F (x) and G(x) are polynomials with integer coefficients such that
F (k)/G(k) is an integer for k = 1, 2, 3, . . ., prove that G(x) divides F (x).

15. Given that ABCDEF is a skew hexagon such that each pair of opposite sides
are equal and parallel. Prove that the midpoints of the six sides are coplanar.
16.

If a, b, c, d are the lengths of sides of a quadrilateral, show that

a
b
c
d
,
,
,
,
(4 + a)
(4
+
c)
(4 + b)
(4 + d)

are possible lengths of sides of another quadrilateral.

17.

Determine the maximum value of the sum of the cosines of the six dihedral
angles of a tetrahedron.

18.
19.

Which is larger

3
( 2 1)1/3

or

Prove that
3 min

p
3

a b
c b
c a
+ + , + +
b
c a a b
c

1/9

p
p
3
2/9 + 3 4/9?

(a + b + c)

1 1 1
+ +
a b
c

where a, b, c are sides of a triangle.

20.
21.

Let = ei/13 . Express

1
1

as a polynomial in with integral coefficients.

Determine all integral solutions of the simultaneous Diophantine equations


x2 + y 2 + z 2 = 2w2 and x4 + y 4 + z 4 = 2w4 .

22.

Prove that if the line joining the incentre to the centroid of a triangle
is parallel to one of the sides of the triangle, then the sides are in arithmetic
progression and, conversely, if the sides of a triangle are in arithmetic progression
then the line joining the incentre to the centroid is parallel to one of the sides of
the triangle.

23.

Determine integral solutions of the Diophantine equation


xy yz
zw wx
+
+
+
=0
x+y y+z
z+w w+x

(joint problem with Emeric Deutsch, Polytechnic University of Brooklyn).

24.

For x, y, z > 0, prove that



x
1
1
1+
,
(i) 1 +
(x + 1)
x(x + 2)

(ii) [(x + y)(x + z)]x [(y + z)(y + x)]y [(z + x)(z + y)]z [4xy]x [4yz]y [4zx]z .

25.

If ABCD is a quadrilateral inscribed in a circle, prove that the four lines


joining each vertex to the nine point centre of the triangle formed by the other
three vertices are concurrent.

26.

How many six digit perfect squares are there each having the property that
if each digit is increased by one, the resulting number is also a perfect square?

27.

Let Vi Wi , i = 1, 2, 3, 4, denote four cevians of a tetrahedron V1 V2 V3 V4 which


are concurrent at an interior point P of the tetrahedron. Prove that
P W1 + P W2 + P W3 + P W4 max Vi Wi longest edge.

28. Determine the radius r of a circle inscribed in a given quadrilateral if the


lengths of successive tangents from the vertices of the quadrilateral to the circle
are a, a, b, b, c, c, d, d, respectively.
29.
30.

Determine the four roots of the equation x4 + 16x 12 = 0.

31.
32.

If 311995 divides a2 + b2 , prove that 311996 divides ab.

Prove that the smallest regular ngon which can be inscribed in a given
regular ngon is one whose vertices are the mid-points of the sides of the given
regular ngon.
Determine the minimum value of
S =

p
p
(a + 1)2 + 2(b 2)2 + (c + 3)2 + (b + 1)2 + 2(c 2)2 + (d + 3)2 ) +
p
p
(c + 1)2 + 2(d 2)2 + (a + 3)2 + (d + 1)2 + 2(a 2)2 + (b + 3)2

where a, b, c, d are any real numbers.

33. A set of 500 real numbers is such that any number in the set is greater than
one-fifth the sum of all the other numbers in the set. Determine the least number
of negative numbers in the set.

34.

Prove that
a+b+c

b2 + c2 a 2 +

p
p
c2 + a 2 b2 + a2 + b2 c2 ,

where a, b, c are sides of a non-obtuse triangle.

35. Determine the extreme values of the area of a triangle ABC given the lengths
of the two altitudes ha , hb and the side BC = a.

36. Determine the maximum area of a triangle ABC given the perimeter p and
the angle A.
37.

Determine the minimum value of


X  a2 + a3 + a4 + a5 1/2
a1

where the sum is cyclic over the positive numbers a1 , a2 , a3 , a4 , a5 .

38.

ABCD and AB 0 C 0 D0 are any two parallelograms in a plane with A opposite


to C and C 0 . Prove that BB 0 , CC 0 and DD0 are possible sides of a triangle.

39.

Determine the maximum value of

S = 4(a4 + b4 + c4 + d4 ) (a2 bc + b2 cd + c2 da + d2 ab) (a2 b + b2 c + c2 d + d2 a) ,


where 1 a, b, c, d 0.

40.
(i)

If a, b, c, d are > 0, prove or disprove the two inequalities:

ab
bc
cd
da
c + d + a + b
2
2
2

a + b + c + d,

(ii) a b + b c + c d + d2 a abc + bcd + cda + dab.

41. Determine all the points P (x, y, z), if any, such that all the points of tangency
of the enveloping (tangent) cone from P to the ellipsoid
b > c), are coplanar.

x2
a2

y2
b2

z2
c2

= 1 (a >

42. Determine whether or not there exists a set of 777 distinct positive integers
such that for every seven of them, their product is divisible by their sum.

43. If R is any non-negative rational approximation to 5, determine an always


better rational approximation.
44.

A sphere of radius R is tangent to each of three concurrent mutually


orthogonal lines. Determine the distance D between the point of concurrence
and the centre of the sphere.

45.

If P (x, y, z, t) is a polynomial in x, y, z, t such that P (x, y, z, t) = 0 for all


real x, y, z, t satisfying x2 + y 2 + z 2 t2 = 0, prove that P (x, y, z, t) is divisible by
x2 + y 2 + z 2 t 2 .

46. From a variable point P on a diameter AB of a given circle of radius r, two


segments P Q and P R are drawn terminating on the circle such that the angles
QP A and RP B are equal to a given angle . Determine the maximum length of
the chord QR.
(rs)!
47. Using that s!(r!)
is an integer, where r, s are positive integers, prove that
s
(rst)!
is an integer for positive integers r, s, t.
t!(s!)t (r!)ts
48.

tan(x + y)
given that
tan x

sin y = 2 sin(2x + y) .

Determine the range of

49.

A, B, C are acute angles such that sin2 A + sin2 B + sin2 C = 2. Prove that
A + B + C < 180 .

50.

Determine the maximum and minimum values of


p
p
a2 cos2 + b2 sin2 + a2 sin2 + b2 cos2 ,

where a and b are given constants.

51.
52.
53.

Determine the range of values of

tan 3x
for x in [0, 2].
tan x

How large can the sum of the angles of a spherical right triangle be?

Let ABC be a spherical triangle


whose mid-points of the sides are A0 , B 0 ,


and C 0 . If B 0 C 0 is a quadrant
, find the maximum value of A0 B 0 + A0 C 0 .
2

54.

Let ABC be a triangle with centroid G. Determine the point P in the plane
of ABC such that AP AG + BP BG + CP CG is a minimum and express this
minimum value in terms of the side lengths of ABC.

KLAMKIN QUICKIES SOLUTIONS

1.

Determine the extreme values of r1 /h1 + r2 /h2 + r3 /h3 + r4 /h4 where h1 , h2 ,


h3 , h4 are the four altitudes of a given tetrahedron T and r1 , r2 , r3 , r4 are the
corresponding signed perpendicular distances from any point in the space of T to
the faces.
Solution. If the face areas and volume of the tetrahedron are F1 , F2 , F3 , F4 , and
V respectively, then
r1 F1 + r2 F2 + r3 F3 + r4 F4 = 3V,
and h1 F1 = h2 F2 = h3 F3 = h4 F4 = 3V . Now eliminating the Fi s, we get
r1 /h1 + r2 /h2 + r3 /h3 + r4 /h4 = 1 (a constant).

2.

Determine the minimum value of the product


P = (1 + x1 + y1 )(1 + x2 + y2 ) . . . (1 + xn + yn )

where xi , yi 0, and x1 x2 . . . xn = y1 y2 . . . yn = an .
Solution. More generally, consider

P = (1 + x1 + y1 + + w1 )(1 + x2 + y2 + + w2 ) . . . (1 + xn + yn + + wn )
where x1 x2 . . . xn = n , y1 y2 . . . yn = n , . . . , w1 w2 . . . wn
xi , yi , . . . , wi 0. Then by H
olders inequality,
n
Y 1/n Y 1/n
Y 1/n o
P 1/n 1 +
xi +
yi + . . . +
wi

n , and

or

P (1 + + + . . . + )n .
In this case = = a, so
P (1 + 2a)n .

3.

Prove that if F (x, y, z) is a concave function of x, y, z, then {F (x, y, z)}2 is


a convex function of x, y, z.
Solution. More generally G(F ) is a convex function where G is a convex decreasing
function. By convexity of G,
G{F (x1 , y1 , z1 )}+(1)G{F (x2 , y2 , z2 )} G{F (x1 , y1 , z1 )+(1)F (x2 , y2 , z2 )}.
By concavity of F ,
F (x1 , y1 , z1 )+(1)F (x2 , y2 , z2 ) F ([x1 +(1)x2 ], [y1 +(1)y2 ], [z1 +(1)z2 ]).
Finally, since G is decreasing,
G{F (x1 , y1 , z1 )} + (1 )G{F (x2 , y2 , z2 )}

G{F ([x1 + (1 )x2 ], [y1 + (1 )y2 ], [z1 + (1 )z2 ])}.

More generally and more precisely, we have the following known result: if F (X)
is a concave function of X = (x1 , x2 , . . . , xn ) and G(y) is a convex decreasing
function of y where y is a real variable and the domain of G contains the range of
F , then G{F (X)} is a convex function of X.

4.

If a, b, c are sides of a given triangle of perimeter p, determine the maximum


values of

(i) (a b)2 + (b c)2 + (c a)2 ,


(ii) |a b| + |b c| + |c a|,

(iii) |a b||b c| + |b c||c a| + |c a||a b|.


P
P
Solution. (i) (a b)2 + (b c)2 + (c a)2 = 2( a2 bc) kp2 .

Let c = 0, so that k 1/2. We now show that k = 1/2 suffices. Here,


2(
reduces to

a2

bc)

1
(a + b + c)2
2

2bc + 2ca + 2ab a2 b2 c2 0.


The LHS is 16 times the square of the area of a triangle of sides

a,

b,

c or

( a + b + c)( a + b c)( a b + c)( a + b + c).


There is equality iff the triangle is degenerate with one side 0.
(ii) |a b| + |b c| + |c a| kp.

Letting c = 0, k 1. To show that k = 1 suffices, assume that a b c, so that


|a b| + |b c| + |c a| = 2a 2c a + b + c

and there is equality iff c = 0.


(iii) |a b| |b c| + |b c| |c a| + |c a| |a b| kp2 .
Letting c = 0, k 1/4. To show that k = 1/4 suffices, let a = y + z, b = z + x,
c = x + y where z y x 0. Our inequality then becomes
|x y| |z y| + |y z| |z x| + |z x| |x y| (x + y + z)2
or
x2 y 2 + z 2 + yz 3zx + xy x2 + y 2 + z 2 + 2yz + 2zx + 2xy
or
2y 2 + 5zx + 1xy + 1yz 0.
There is equality iff x = y = 0 or equivalently, a = b, and c = 0.

5. If A, B, C are three dihedral angles of a trihedral angle, show that sin A, sin B,
sin C satisfy the triangle inequality.
Solution. Let a, b, c be the face angles of the trihedral angle opposite to A, B, C
respectively. Since
sin a
sin b
sin c
=
=
sin A
sin B
sin C
by the Law of Sines for spherical triangles, it suffices to show that sin b + sin c >
sin a, or
1
1
1
1
2 sin (b + c) cos (b c) > 2 sin a cos a,
2
2
2
2
for any labelling of the angles. We now use the following properties of a, b, c:
(i) they satisfy the triangle inequality, (ii) 0 < a + b + c < 2.
Hence, cos 12 (b c) > cos 21 a. To complete the proof, we show that
1
1
sin (b + c) > sin a.
2
2
This follows immediately if b + c ; if b + c > , then




1
1
1
b+c
a
sin (b + c) = sin (b + c) > sin a
since
>
.
2
2
2
2
2
Comment: More generally, if a1 , a2 , . . . , an are the sides of a spherical n-gon
(convex), it then follows by induction over n that
sin a1 + sin a2 + + sin an > 2 sin ai ,

i = 1, 2, . . . , n.

It also follows by induction that


| sin a1 | + | sin a2 | + + | sin an | > | sin(a1 + a2 + + an )|
for any angles a1 , a2 , . . . , an .

6.

Are there any integral solutions (x, y, z) of the Diophantine equation


(x y z)3 = 27xyz

other than (a, a, a) or such that xyz = 0?


Solution. Let x = u3 , y = v 3 , z = w3 , so that u3 v 3 w3 = 3uvw or equivalently
(u v w)((u + v)2 + (u + w)2 + (v w)2 ) = 0.
Hence an infinite class of non-trivial solutions is given by
x = (v + w)3 ,

y = v3,

z = w3 .

Whether or not there are any other solutions is an open problem.

10

7.

Does the Diophantine equation


(x y z)(x y + z)(x + y z) = 8xyz

have an infinite number of relatively prime solutions?


Solution. By inspection, we have the trivial solutions
(x, y, z) = (1, 1, 0)
and permutations thereof.
equations

For other solutions, note that each of the three

x y z = 2 yz,

x y + z = 2 xz,

x + y z = 2 xy

is satisfied by x = y + z. Consequently, we also have the infinite set of


solutions
y = m2 , z = n2 , x = (m + n)2 where (m, n) = 1.
It is an open problem whether or not there are any other infinite sets of relatively
prime solutions.

8.

It is an easy result using calculus that if a polynomial P (x) is divisible by


its derivative P 0 (x), then P (x) must be of the form a(x r)n . Starting from the
known result that
P 0 (x) X 1
=
P (x)
x ri

where the sum is over all the zeros ri of P (x) counting multiplicities, give a
non-calculus proof of the above result.

Solution. Since P 0 (x) is of degree one less than that of P (x),


X 1 
P 0 (x)
1
=
=
.
P (x)
a(x r)
x ri
Now letting x any ri it follows that r = ri . Hence all the zeros of P (x) must
be the same.

9.

Solve the simultaneous equations


x2 (y + z) = 1,

y 2 (z + x) = 8,

z 2 (x + y) = 13.

Solution. More generally we can replace the constants 1, 8, 13 by a3 , b3 , c3 ,


respectively. Then by addition of the three equations and by multiplication of the
three equations, we respectively get
X
x2 y = a 3 + b 3 + c 3 ,
h
i
X
(xyz)2 2xyz +
x2 y = (abc)3 ,

11

where the sums are symmetric over x, y, z. Hence,


2t3 + t2 (a3 + b3 + c3 ) = (abc)3

(1)

where t = xyz. In terms of t, the original equations can be rewritten as


a3
1 1
= 0,
tx y z

1 b3
1
+
= 0,
x
ty z

1 1 c3
+
= 0.
x
y
tz

These latter homogeneous equations are consistent since the eliminant is equation
(1). Solving the last two equations for y and z, we get
y=

x(b1 c1 1)
,
c1 + 1

z=

x(b1 c1 2)
b1 + 1

where b1 = b3 /t, c1 = c3 /t. On substituting back in x2 (y + z) = a3 , we obtain x3


and then x, y, z.

10.

Determine the area of a triangle of sides a, b, c and semiperimeter s if


(s b)(s c) = a/h,

(s c)(s a) = b/k,

(s a)(s b) = c/l,

where h, k, l, are consistent given constants.


Solution.
h=

a
1
1
=
+
,
(s b)(s c)
(s b) (s c)

k =

1
1
+
,
(s c) (s a)

l =

1
1
+
.
(s a) (s b)

Hence, h, k, l must satisfy the triangle inequality. Letting 2s0 = h+k +l, it follows
by addition that
1
1
1
s0 =
+
+
(s a) (s b) (s c)

and then

sa=

(s0

1
,
h)

sb=

(s0

1
,
k)

sc=

(s0

1
.
l)

Adding the latter three equations, we get


s=

1
1
1
+
+
.
(s0 h) (s0 k) (s0 l)

Finally, the area of the triangle is given by


(
= {s(s a)(s b)(s c)}

1/2

1
1
1
(s0 h) + (s0 k) + (s0 l)
(s0 h)(s0 k)(s0 l)

)1/2

12

11.

Prove that
3(x2 y + y 2 z + z 2 x)(xy 2 + yz 2 + zx2 ) xyz(x + y + z)2

where x, y, z 0.

Solution. By Cauchys inequality

(x2 y + y 2 z = z 2 x)(zx2 + xy 2 + yz 2 ) (x2 yz + y 2 zx + z 2 xy)2 .


Hence it suffices to show that


(x3/2 + y 3/2 + z 3/2


3

2

(x + y + z)
3

3

But this follows immediately from the power mean inequality. There is equality
iff x = y = z.

12.

Determine all integral solutions of the Diophantine equation


(x8 + y 8 + z 8 ) = 2(x16 + y 16 + z 16 ).

Solution. More generally one can find all integral solutions of


(x2n + y 2n + z 2n )2 = 2(x4n + y 4n + z 4n ),

(2)

where n is a positive integer provided Fermats equation xn + y n = z n does not


have any integer solutions for particular values of n > 2 chosen.
Equation (2) can be rewritten as
(xn + y n + z n )(y n + z n xn )(z n + xn y n )(xn + y n z n ) = 0.

(3)

The trivial solutions occur for (x, y, z) = (a, a, 0) and permutations thereof.
For n = 1, any factor of the left hand side of (3) can be zero.
For n = 2, (x, y, z) can be the sides of any integral right triangle (2mn, m2 n2 ,
m2 + n2 ) in any order.
Since Fermats equation is at least known not to have any non-trivial solutions for
all n > 2 and < 100 and integral multiples thereof, there are not any non-trivial
solutions for at least these cases.

13.

Determine all the roots of the quintic equation


31x5 + 165x4 + 310x3 + 330x2 + 155x + 33 = 0.

Solution. Since the equation can be rewritten as (x 1)5 = 32(x + 1)5 ,


x1
= 2 r ,
x+1

r = 0, 1, 2, 3, 4

13

where is a primitive 5th root of unity. Hence,


x=

1 + 2 r
,
1 2 r

r = 0, 1, 2, 3, 4.

More generally the equation


ax6 + 5bcx4 + 10ac2 x3 + 10bc3x2 + 5ac4 x + bc5 = 0
is the same as
(b a)(x c)5 = (b + a)(x + c)5 .

14.

If F (x) and G(x) are polynomials with integer coefficients such that
F (k)/G(k) is an integer for k = 1, 2, 3 . . . , prove that G(x) divides F (x).
Solution.
By taking k sufficiently large it follows that the degree of F
is the degree of G. Then by the remainder theorem,
F (x)
Q(x) R(x)
=
+
G(x)
a
G(x)

where Q(x) is an integral polynomial, a is an integer, and R(x) is a polynomial


whose degree is less than that of G(x). Now R(x) must identically vanish otherwise
by taking k sufficiently large, we can make R(k)/G(k) arbitrarily small and this
cannot add with Q(k)/a to be an integer.

15. Given that ABCDEF is a skew hexagon such that each pair of opposite sides
are equal and parallel. Prove that the midpoints of the six sides are coplanar.
Solution. Since each pair of opposite sides form a parallelogram whose diagonals
bisect each other, all three different diagonals are concurrent say at point P . We
now let A, B, C, A, B, C be vectors from P to A, B, C, D, E, F , respectively.
The successive midpoints (multiplied by 2) are given by
A + B,

B + C,

C A,

A B,

B C,

C + A

and which incidentally form another centrosymmetric hexagon. It is enough now


to note that (A + B) (B + C) + (C A) = 0.

16.

If a, b, c, d are the lengths of sides of a quadrilateral, show that

a
b
c
d
,
,
,
,
(4 + a)
(4
+
c)
(4 + b)
(4 + d)

are possible lengths of sides of another quadrilateral.


Solution. More generally one can show that if a1 , a2 , . . . , an are the lengths of sides
of an n-gon, then F (a1 ), F (a2 ), . . . , F (an ) are possible lengths of sides of another
n-gon where F (x) is an increasing concave function of x for x 0 and F (0) = 0.
If a1 is the largest of the ai s, then it suffices to show that

F (a2 ) + F (a3 ) + + F (an ) F (a1 ).

14

By the majorization inequality, we have


F (a2 ) + F (a3 ) + + F (an ) F (a2 + a3 + + an ) + (n 2)F (0).
Finally, F (a2 + a3 + + an ) F (a1 ).

Some admissable functions are


F (x) = x

and

x
k 2 + x

for 0 < < 1,

2
x
, 1 ek x , tanh x.
(x + k 2 )

17.

Determine the maximum value of the sum of the cosines of the six dihedral
angles of a tetrahedron.
Solution. Let A, B, C, D be unit outward vectors normal to the faces of a
tetrahedron ABCD. Then
(xA + yB + zC + wD)2 0.
Expanding out and noting that A B = cos CD (here CD denotes the dihedral
angle of which the side CD is an edge), etc., we get
x2 + y 2 + z 2 + w2 2xy cos CD + 2xz cos BD + 2xw cos BC

(1)

+2yz cos AD + 2yw cos AC + 2zw cos AB.

Setting x = y = z = w, we get that the sum of the cosines of the 6 dihedral angles
is 2. There is equality iff A + B + C + D = 0. Since as known
Fa A + F b B + F c C + F d D = 0
where Fa denotes the area of the face of the tetrahedron opposite A, etc., it follows
that there is equality iff the four faces have equal area or that the tetrahedron is
isosceles.
Comment. In a similar fashion one can extend inequality (1) to n dimensions
and then show that the sum of the cosines of the n(n + 1)/2 dihedral angles
of an n-dimensional simplex is (n + 1)/2. Here the dihedral angles are the
angles between pairs of (n 1)-dimensional faces and there is equality iff all the
(n 1)-dimensional faces have the same volume.

18.

Which is larger

3
( 2 1)1/3

or

p
3

1/9

p
p
3
2/9 + 3 4/9?

Solution. That they are equal is an identity of Ramanujan.


p
p
Letting x = 3 1/3 and y = 3 2/3, it suffices to show that

3
(x + y)( 2 1)1/3 = x3 + y 3 = 1,
or equivalently that

3
3
( 2 + 1)3 ( 2 1) = 3,

15

which follows by expanding out the left hand side.


For other related radical identities of Ramanujan, see Susan Landau,
How to tangle with a nested radical, Math.
Intelligencer, 16 (1994),
pp. 4954.

19.

Prove that
3 min

a b
c b
c a
+ + , + +
b
c a a b
c

(a + b + c)

1 1 1
+ +
a
b
c

where a, b, c are sides of a triangle.


Solution. Each of the inequalities




a b
c
1 1 1
3
+ +
(a + b + c)
+ +
,
b
c a
a b
c




b
c a
1 1 1
3
+ +
(a + b + c)
+ +
,
a b
c
a b
c
follow from their equivalent forms (which follow by expansion):
(b + a c)(c a)2 + (c + b a)(a b)2 + (a + c b)(b c)2 0,
(b + c a)(c a)2 + (c + a b)(a b)2 + (a + b c)(b c)2 0.

20.

Let = ei/13 . Express

1
1

as a polynomial in with integral coefficients.

Solution. We have
2
(1 13 )
=
= 1 + + 2 + + 12 ,
(1 )
(1 )
(1 + 13 )
= 1 + 2 + 12 .
0=
(1 + )
Adding or subtracting, we get
1
= 1 + 2 + 4 + + 12
(1 )
= + 3 + + 11 .

More generally, if = ei/(2n+1) ,


1
= 1 + 2 + 4 + + 2n .
(1 )

21.

Determine all integral solutions of the simultaneous Diophantine equations


x2 + y 2 + z 2 = 2w2 and x4 + y 4 + z 4 = 2w4 .
Solution. Eliminating w we get
2y 2 z 2 + 2z 2 x2 + 2x2 y 2 x4 y 4 z 4 = 0

16

or
(x + y + z)(y + z x)(z + x y)(x + y z) = 0,
so that in general we can take z = x + y. Note that if (x, y, z, w) is a solution, so
is (x, y, z, w) and permutations of the x, y, z. Substituting back, we get
x2 + xy + y 2 = w2 .
Since (x, y, w) = (1, 1, 1) is one solution, the general solution is obtained by
the method of Desboves, that is, we set x = r + p, y = r + q and w = r.
2
+pq+q 2 )
This gives r = (p (qp)
. On rationalizing the solutions (since the equation is
homogeneous), we get
x = p2 + pq + q 2 + p(q p) = q 2 + 2pq,

y = p2 + pq + q 2 q(q p) = p2 + 2pq,
w = p2 + pq + q 2 ,
z = q 2 p2 .

22.

Prove that if the line joining the incentre to the centroid of a triangle
is parallel to one of the sides of the triangle, then the sides are in arithmetic
progression and, conversely, if the sides of a triangle are in arithmetic progression
then the line joining the incentre to the centroid is parallel to one of the sides of
the triangle.
Solution. Let A, B, C denote vectors to the respective vertices A, B, C of the
triangle from a point outside the plane of the triangle. Then the incentre I and
the centroid G have the respective vector representations I and G, where
I=

(aA + bB + cC)
,
(a + b + c)

G=

(A + B + C)
,
3

(where a, b, c are sides of the triangle). If G I = k(A B), then by expanding


out
(b + c 2a k 0 )A + (a + c 2b + k 0 )B + (a + b 2c)C = 0,
where k 0 = 3k(a + b + c). Since A, B, C are linearly independent, the coefficient
of C must vanish so that the sides are in arithmetic progression. Also then k 0 =
b + c 2a = 2b a c.
Conversely, if 2c = a + b, then G I =
side AB.

3(AB)(ba)
,
6(a+b+c)

so that GI is parallel to the

17

23.

Determine integral solutions of the Diophantine equation


xy yz
zw wx
+
+
+
=0
x+y y+z
z+w w+x

(joint problem with Emeric Deutsch, Polytechnic University of Brooklyn).


Solution. It follows by inspection that x = z and y = w are two solutions. To find
the remaining solution(s), we multiply the given equation by the least common
denominator to give
P (x, y, z, w) = 0,
where P is the 4th degree polynomial in x, y, z, w which is skew symmetric in x
and z and also in y and w. Hence,
P (x, y, z, w) = (x z)(y w)Q(x, y, z, w),
where Q is a quadratic polynomial. On calculating the coefficient of x2 in P , we
get 2z(y w). Similarly the coefficient of y 2 is 2w(x z), so that
P (x, y, z, w) = 2(x z)(y w)(xz yw).
Hence, the third and remaining solution is given by xz = yw.

24.

For x, y, z > 0, prove that



x
1
1
(i) 1 +
1+
,
(x + 1)
x(x + 2)

(ii) [(x + y)(x + z)]x [(y + z)(y + x)]y [(z + x)(z + y)]z [4xy]x [4yz]y [4zx]z .

Solution. Both inequalities will follow by a judicious application of the weighted


arithmetic-geometric mean inequality (WA.M.G.M.) which for three weights is


au + bv + cw
u v w
a+b+c
a b

a+b+c

where a, b, c, u, v, w 0.

(i) The inequality can be rewritten in the more attractive form



x 
x+1
1
1
1+
1+
,
x
x+1

and which now follows by the WA.M.G.M.


 )x+1 

x (
x+1
1 + x 1 + x1
1
1
1+

= 1+
.
x
1+x
x+1
(ii) Also, the inequality here can be rewritten in the more attractive form

z+x 
x+y 
y+z
2x
2y
2z
1.
z+x
x+y
y+z

18

But this follows by applying the WA.M.G.M. to




X
2x X
1=
[z + x]
[z + x].
z+x

25.

If ABCD is a quadrilateral inscribed in a circle, prove that the four lines


joining each vertex to the nine point centre of the triangle formed by the other
three vertices are concurrent.

Solution. The given result still holds if we replace the nine point centres by either
the orthocentres or the centroids.
A vector representation is particularly a
` propos here, since (with the circumcentre
O as an origin and F denoting the vector from O to any point F ) the orthocentre
Ha , the nine point centre Na , the centroid Ga of 4BCD are given simply by
Ha = B + C + D, Na = (B + C + D)/2, Ga = (B + C + D)/3, respectively, and
similarly for the other three triangles. Since the proofs for each of the three cases
are practically identical, we just give the one for the orthocentres. The vector
equation of the line La joining A to Ha is given by La = A + a [B + C + D A]
where a is a real parameter. By letting a = 1/2, one point on the line is
[A + B + C + D]/2 and similarly this point is on the other three lines. For the
nine point centres, the point of concurrency will be 2[A + B + C + D]/3, while
for the centroids, the point of concurrency will be 3[A + B + C + D]/4.

26.

How many six digit perfect squares are there each having the property that
if each digit is increased by one, the resulting number is also a perfect square?

Solution. If the six digit square is given by


m2 = a 105 + b 104 + c 103 + d 102 + e 10 + f,
then
n2 = (a + 1) 105 + (b + 1) 104 + (c + 1) 103 + (d + 1) 102 + (e + 1) 10 + (d + 1),
so that
n2 m2 = 111, 111 = (111)(1, 001) = (3 37)(7 11 13).
Hence,
n + m = di

and n m = 111, 111/di

where di is one of the divisors of 111, 111. Since 111, 111 is a product of five primes
it has 32 different divisors. But since we must have di > 111, 111/di, there are at
most 16 solutions given by the form lm = 21 (di 111, 111/di). Then since m2 is a
six digit number, we must have

632.46 200 10 < 2m < 2, 000.


On checking the various divisors, there are four solutions. One of them corresponds
to di = 3 13 37 = 1, 443 so that m = 21 (1, 443 7 11) = 683 and m2 = 466, 489.

19

Then, 466, 489 + 111, 111 = 577, 600 = 7602 . The others are given by the table
m
m2
317 100, 489
565 319, 225
445 198, 025

di
3 7 37 = 777
3 11 37 = 1, 221
7 11 13 = 1, 001

n2
n
211, 600 460
430, 336 656
309, 136 556

27.

Let vi wi , i = 1, 2, 3, 4, denote four cevians of a tetrahedron v1 v2 v3 v4 which


are concurrent at an interior point P of the tetrahedron. Prove that
pw1 + pw2 + pw3 + pw4 max vi wi longest edge.

Solution. We choose an origin, o, outside of the space of the tetrahedron and use
the set of 4 linearly independent vectors Vi = ovi as a basis. Also the vector
from o to any point q will be denoted by Q. The interior
point p is then given by
P
P = x1 V1 + x2 V2 + x3 V3 + x4 V4 where xi > 0 and i xi = 1. It now follows that
i Vi
Wi = Px
(for other properties of concurrent cevians via vectors, see [1987:
1xi
274275]) and then that






xi (P Vi ) X Vj Vi
P x i Vi




,

pwi =
P =
= xi
xj

1 xi
1 xi
1

x
i

j

Summing







P x i Vi
P V i X Vj V i





.
vi wi =
Vi =
=
xj
1 xi
1 xi j
1 xi

X
i



X X Vj Vi X
xi
=
pwi =
xj
xi (vi wi ) max vi wi ,

i
1 xi
i
j
i

and with equality only if vi wi is constant. Also,

X  xj 
vi wi
max |Vr Vi | = max |Vr Vi |.
r
r
1 xi
j6=i

Finally,

pwi max vi wi max |Vr Vs |.


i

r,s

Comment: In a similar fashion, it can be shown that the result generalizes


to n-dimensional simplexes. The results for triangles are due to Paul Erd
os,
Amer. Math. Monthly, Problem 3746, 1937, p. 400; Problem 3848, 1940, p. 575.

20

28. Determine the radius r of a circle inscribed in a given quadrilateral if the


lengths of successive tangents from the vertices of the quadrilateral to the circle
are a, a, b, b, c, c, d, d, respectively.
Solution. Let 2A, 2B, 2C, 2D denote the angles between successive pairs of radii
vectors to the points of tangency and let r be the inradius. Then
r=

a
b
c
d
=
=
=
.
tan A
tan B
tan C
tan D

Also, since A + B + C + D = , we have tan(A + B) = tan(C + D) = 0, or


tan A + tan B
tan C + tan D
+
= 0,
1 tan A tan B
1 tan C tan D
so that
r(a + b) r(c + d)
+ 2
= 0.
r2 ab
r cd
Finally,
r2 =

29.

abc + bcd + cda + dab


.
a+b+c+d

Determine the four roots of the equation x4 + 16x 12 = 0.

Solution. Since
x4 + 16x 12 = (x2 + 2)2 4(x 2)2 = (x2 + 2x 2)(x2 2x + 6) = 0,
the four roots are 1

30.

3 and 1 i 5.

Prove that the smallest regular n-gon which can be inscribed in a given
regular n-gon is one whose vertices are the midpoints of the sides of the given
regular n-gon.
Solution. The circumcircle of the inscribed regular n-gon must intersect each side
of the given regular n-gon. The smallest that such a circle can be is the inscribed
circle of the given n-gon, and it touches each of its sides at its midpoints.

31.

If 311995 divides a2 + b2 , prove that 311996 divides ab.

Solution. If one calculates 12 , 22 , . . . , 302 mod 31 one finds that the sum of no two
of these equals 0 mod 31. Hence, a = 31a1 and b = 31b1 so that 311993 divides
a21 + b21 . Then, a1 = 31a2 and b1 = 31b2 . Continuing in this fashion (with p = 31),
we must have a = p998 m and b = p998 n so that ab is divisible by p1996 .
More generally, if a prime p = 4k + 3 divides a2 + b2 , then both a and b must be
divisible by p. This follows from the result that a natural n is the sum of squares
of two relatively prime natural numbers if and only if n is divisible neither by 4 nor
by a natural number of the form 4k + 3 (see J.W. Sierpi
nski, Elementary Theory
of Numbers, Hafner, NY, 1964, p. 170).

21

32.

Determine the minimum value of


S=

p
p
(a + 1)2 + 2(b 2)2 + (c + 3)2 + (b + 1)2 + 2(c 2)2 + (d + 3)2 )
p
p
+ (c + 1)2 + 2(d 2)2 + (a + 3)2 + (d + 1)2 + 2(a 2)2 + (b + 3)2

where a, b, c, d are any real numbers.

Solution. Applying Minkowskis inequality,


S

(4 + s)2 + 2(s 8)2 + (s + 12)2 =

4s2 + 288

where s = a + b + c + d. Consequently, min S = 12 2 and is taken on for


a = b = c = d = 0.

33. A set of 500 real numbers is such that any number in the set is greater than
one-fifth the sum of all the other numbers in the set. Determine the least number
of negative numbers in the set.
Solution. Letting a1 , a2 , a3 , . . . denote the numbers of the set and S the sum of all
the numbers in the set, we have
a1 >

S a1
,
5

a2 >

S a2
,
5

...,

a6 >

S a6
.
5

Adding, we get 0 > S a1 a2 a6 so that if there were six or less negative


numbers in the set, the right hand side of the inequality could be positive. Hence,
there must be at least seven negative numbers.
Comment. This problem where the 5 is replaced by 1 is due to Mark
Kantrowitz, CarnegieMellon University.

34.

Prove that
a+b+c

b2 + c2 a 2 +

p
p
c2 + a 2 b2 + a2 + b2 c2 ,

where a, b, c are sides of a non-obtuse triangle.


Solution. By the power mean inequality

p
p
b2 + c2 a2 + c2 + a2 b2 2c ,

and similarly two other such inequalities. Then, adding, we get the desired result.

35. Determine the extreme values of the area of a triangle ABC, given the lengths
of the two altitudes hc , hb . [Correction to question made.]
Solution. Let D and E be the feet of the altitudes hb and hc . Then by the Law
hb
hc
of Sines applied to triangles ABD and ACE, c = sin
A and b = sin A . Twice the
hb hc
hb hc
area is given by 2[ABC] = sin A . Hence, the minimum area is 2 occuring for
A = 2 . Also, by letting A approach , the area becomes unbounded. In this case
a would be arbitrarily large.

22

36. Determine the maximum area of a triangle ABC given the perimeter p and
the angle A.
Solution. Since 2[ABC] = bc sin A, we have to maximize bc subject to
p = a + b + c, and a2 = b2 + c2 2bc cos A .

Since p a = b + c 2 bc, bc will be a maximum when b = c regardless of the


value of a. Thus, we have
p = a + 2b

and

a2 = 2b2 2b2 cos A .

Then (p 2b)2 = 2b2 2b2 cos A. Solving for b:


b =

p 1 + sin A
2
2 cos2

so that
max[ABC] =

37.

p2 tan A
2

A
2

1 + sin A
2

4 cos2

A
2

2

Determine the minimum value of


X  (a2 + a3 + a4 + a5 ) 1/2
a1

where the sum is cyclic over the positive numbers a1 , a2 , a3 , a4 , a5 .


Solution. Applying the AMGM Inequality to each term of the sum, the given
sum is greater than or equal to
X  a2 a3 a4 a5 1/8
2
a41
(where again the sum is cyclic). Finally applying the AMGM Inequality again,
the latter sum is greater than or equal to 10. There is equality in the given
inequality if and only if the ai s are equal. In a similar fashion it follows that if
we increase the number of variables to n + 1 and change the 1/2 power to any
positive number p, the minimum here would be (n + 1)p .

38.

ABCD and AB 0 C 0 D0 are any two parallelograms in a plane with A opposite


to C and C 0 . Prove that BB 0 , CC 0 and DD0 are possible sides of a triangle.
Solution. Let the vectors from A to B and A to D be denoted by U1 and U2 , and
the vectors from A to B 0 and A to D0 be denoted by V1 and V2 . Then
BB0 = V1 U1 , DD0 = V2 U2 , and CC0 = V1 + V2 U1 U2 ,
so that CC0 = BB0 +DD0 . The rest follows from the triangle inequality |PQ|
|P| + |Q| and with equality only if P and Q have the same direction.

23

Remarks: Christopher J. Bradley of Clifton College, Bristol, UK, also submitted


solutions for the quickies, noting a problem with problem 2. He points out that
he gave problem 1 to a group of students in 1988.

39.

Determine the maximum value of

S = 4(a4 + b4 + c4 + d4 ) (a2 bc + b2 cd + c2 da + d2 ab) (a2 b + b2 c + c2 d + d2 a) ,


where 1 a, b, c, d 0.
Solution.

S 4(a2 + b2 + c2 + d2 ) (a2 b2 c2 + b2 c2 d2 + c2 d2 a2 + d2 a2 b2 )
(a2 b2 + b2 c2 + c2 d2 + d2 a2 ).

Since the expression on the right hand side is linear in a2 , b2 , c2 ,


and d2 , it takes on its maximum at the endpoints 0, 1 for each variable.
By inspection, Smax = 9 and is taken on for a = b = c = 1 and d = 0.

40.
(i)

If a, b, c, d are > 0, prove or disprove the two inequalities:

ab
bc
cd
da
c + d + a + b
2
2
2

a + b + c + d,

(ii) a b + b c + c d + d2 a abc + bcd + cda + dab.

Solution. Neither inequality is valid.

(i) Just consider the case: b = 2, c = 5, d = 1 and a is very large.


(ii) Just consider the case: a = 2, b = 1, c = 8 and d is very small.

41. Determine all the points P (x, y, z), if any, such that all the points
of tangency of the enveloping (tangent) cone from P to the ellipsoid
y2
x2
z2
a2 + b2 + c2 = 1 (a > b > c), are coplanar.
Solution. Consider the affine transformation x0 = xa , y 0 = yb , z 0 = zc which takes
the ellipsoid into a sphere. Under this transformation, lines go to lines, planes go
to planes, and tangency is preserved. Consequently, any enveloping cone of the
ellipsoid goes into an enveloping cone of the sphere and which by symmetry is a
right circular one and its points of tangency are a circle (coplanar) of the sphere.
Thus, P can be any exterior point of the ellipsoid.

42. Determine whether or not there exists a set of 777 distinct positive integers
such that for every seven of them, their product is divisible by their sum.
Solution. Just take any 777 distinct positive integers and multiply each one by the
product of the sums of every 7 of them.

43. If R is any non-negative rational approximation to 5, determine an always


better rational approximation.
aR+b
Solution. Assuming the better approximation has the form cR+d
where a, b, c, d
are rational, we must satisfy

aR + b

< |R 5| .

5
(2)
cR + d

5+b
If R 5, the left hand side must 0. Thus, we must have ca5+d
= 5, so
that d = a and b = 5c. Then substituting these values in (2) and dividing both

24

sides by the common factor |R

5|, we get

|cR + a| > |c 5 a|

and which can easily be satisfied by letting a = 2 and c = 1. Finally, our better
approximation is 2R+5
R+2 .

44.

A sphere of radius R is tangent to each of three concurrent mutually


orthogonal lines. Determine the distance D between the point of concurrence
and the centre of the sphere.
Solution. Let the three lines be the x, y, and zaxes of a rectilinear coordinate
system and the equation of the sphere be (x a)2 + (y b)2 + (z c)2 = R2 . The
required distance squared is a2 + b2 + c2 . Since the distance from the centre of the
sphere to each of the lines is R, we have
R 2 = b2 + c2 = c2 + a2 = a2 + b2 .
Hence, D2 = 3R2 /2.

45.If P (x, y, z, t) is a polynomial in x, y, z, t such that P (x, y, z, t) = 0 for all

real x, y, z, t satisfying x2 + y 2 + z 2 t2 = 0, prove that P (x, y, z, t) is divisible by


x2 + y 2 + z 2 t 2 .
Solution. By the Remainder Theorem,
P (x, y, z, t) = (x2 + y 2 + z 2 t2 )Q(x, y, z, t) + R(x, y, z)t + S(x, y, z)
where Q, R and S are polynomials.
Now, letting t be successively
(x2 + y 2 + z 2 )1/2 , it follows that R = S = 0.

46. From a variable point P on a diameter AB of a given circle of radius r, two


segments P Q and P R are drawn terminating on the circle such that the angles
QP A and RP B are equal to a given angle . Determine the maximum length of
the chord QR.

Solution. Extend the chords QP and RP to intersect the circle again at points Q0
and R0 . It now follows that the arcs QR and Q0 R0 are congruent and thus, their
measures are 2. Then if O is the centre, triangle OQR is isosceles whose
vertex angle is also 2. Hence, QR = 2r cos , which is the same for all P .
(rs)!
47. Using that s!(r!)
is an integer, where r, s are positive integers, prove that
s
(rst)!
is an integer for positive integers r, s, t.
t!(s!)t (r!)ts
Solution. It follows from the given relation that both
(r(st))!
(st)!(r!)st

and

are integers. Now just multiply them together.

(st)!
t!(s!)t

25

48.

tan(x + y)
given that
tan x

sin y = 2 sin(2x + y) .

Determine the range of

Solution. Since
sin y
sin(2x + y)

so that

tan(x + y)
tan x

sin(x + y) cos x cos(x + y) sin x


sin(x + y) cos x + cos(x + y) sin x

tan(x + y) tan x
=
= 2,
tan(x + y) + tan x

= 3 2 2 = a constant.

49.

A, B, C are acute angles such that sin2 A + sin2 B + sin2 C = 2. Prove that
A + B + C < 180 .
Solution. Equivalently, cos2 A + cos2 B + cos2 C = 1. Then, using the identity for
arbitrary angles A, B, C,
cos2 A + cos2 B + cos2 C 1 + 2 cos A cos B cos C
= cos(S) cos(S A) cos(S B) cos(S C) ,

where 2S = A + B + C, we have
cos A cos B cos C = 2 cos(S) cos(S A) cos(S B) cos(S C) .
Since the left-hand-side is positive, it follows that 2S < .

50.

Determine the maximum and minimum values of


p
p
a2 cos2 + b2 sin2 + a2 sin2 + b2 cos2

where a and b are given constants.


p
p
Solution. Let S =
a2 cos2 + b2 sin2 + a2 sin2 + b2 cos2 . Squaring we
get,
q
S 2 = a2 + b2 + 2

(a4 + b4 ) sin2 cos2 + a2 b2 (cos4 + sin4 ) .

Since cos4 + sin4 = 1 2 sin2 cos2 , the expression inside the radical can be
written as
(a2 b2 )2 (sin2 2)
+ a 2 b2 .
4
Hence, the maximum and minimum are taken on for = 4 and 0, respectively,
p
giving Smax = 2(a2 + b2 ) and Smin = a + b.

51.

Determine the range of values of

Solution. Expanding out:


(tan 3x)
=
tan x

tan 3x
tan x

1+

for x in [0, 2].

8
(13 tan2 x)

Since tan x lies in the interval [0, ), the range consists of all values

1
3

and 3.

26

52.

How large can the sum of the angles of a spherical right triangle be?

Solution. Consider a lune with angle 2 . Now draw a very small arc across it near
one of its vertices producing a right triangle of area nearly equal to (for a unit
sphere). Hence the spherical excess A + B + C = . Hence the sum must
be less than 2.

53.

Let ABC be a spherical triangle whose mid-points of the sides are A0 , B 0 ,


and C 0 . If B 0 C 0 is a quadrant 2 , find the maximum value of A0 B 0 + A0 C 0 .

Solution. Let A , B , C be unit vectors from the centre of the sphere to the

A +B
vertices A, B, C of the spherical triangle. Then C 0 and B 0 are given by

A +C
and

.
A + C

A + B



 


We now have that cos B 0 C 0 = 0 = A + B A + C
=

1 + cos a + cos b + cos c where a, b, c are the sides of ABC. Since this is symmetric

in A, B, C, A0 C 0 = A0 B 0 = , so that A0 C 0 + A0 B 0 = the constant .


2

54.

Let ABC be a triangle with centroid G. Determine the point P in the plane
of ABC such that AP AG + BP BG + CP CG is a minimum and express this
minimum value in terms of the side lengths of ABC.

Solution. Let A , B , C , and P be vectors from G to A, B, C, and P , respectively.


Then
AP AG
+
BG + CP CG

BP









= A A P + B B P + C C P










A A P +B B P +C C P

2 
2 
2




=
A
+ B
+ C
P A +B +C

2 
2 
2

=
A
+ B
+ C
since A + B + C = 0 .
4(m2 + m2 + m2 )

(a2 + b2 + c2 )

a
c
b
=
,
Hence the minimum is
9
3
attained when P coincides with G (ma is the median from A, etc.).

and

is

Comment. This was a short-listed problem for the 2001 IMO. It was eliminated
after I gave the above Quickie solution.

27

Combinatorics and Number Theory

429.

[1979 : 77-8] Proposed by Murray S. Klamkin, University of Alberta,


Edmonton, Alberta, and Andy Liu, University of Alberta, Edmonton, Alberta.

On a 2n 2n board we place n 1 polyominoes (each covering exactly n


unit squares of the board) until no more n 1 polyominoes can be accommodated.
What is the maximum number of squares that can be left vacant?
This problem generalizes CRUX 282 [1978 : 114].
Solution by Murray S. Klamkin and Andy Liu, University of Alberta,
Edmonton, Alberta. [1980 : 51]
..........................................................................................................................................................................................................................
..
..
..
..
..
..
..
..
..
..
..
..
..
..
...
...
..
...
..
...
..
..
..
..
..
..
...................................................................................
...
...
...................................................................................
...
..
.
.
.
.
.
.
.
.
.
.
.
.
.
.
.
.
.
.
.
.
.
...
.
.
....
....
....
....
....
....
....
....
....
....
....
..
....
...
...
................................................................................
...
...............................................................................
...
...
...
...
...
...
...
...
...
...
...
...
...
...
...
...
...
...
...
..
...
....
....
....
....
....
..
..
...............................................................................
..
..............................................................................
...
...
...
...
...
...
...
...
...
...
...
...
...
...
...
...
...
...
...
...
...
...
...
...
...
...
...............................................................................
....
....
................................................................................
....
...
..
..
..
..
...
...
...
...
...
...
...
...
...
...
...
...
...
...
...
...
...
...
...
..
...
...
...
................................................................................
...
..............................................................................
.
.
.
.
.
...
.
..
..
..
..
..
.
.
.
.
.
...
.
.
.
.
.
.
.
.
.
.
.
.
..
.
.
.
.
.
.
.
.
.
.
.
.
.
.
.
.
...
.
.
.
.
.
.
.
.
.
.
....
....
....
....
....
...
...
....
....................................................................................
..
..
..
..
..
...
...
...
...
...
...
...
...
...............................................................................
...
...
...
...
...
...
...
...
...
...
...
...
...
...
...
...
...
...
...
...
...
...
...
...
...
...
...
...
...
...
...
...
...
...
...
...
...
...
...
...
...
...
....................................................................................
..
..
..
..
..
...
...
...
...
...
...
...
...
...
...
...
...
...
...
...
...
...
...
...
...
...
.
.
.
.
.
.
.........................................................................
...
.
.
.
.
.
.
...
.
....
....
....
....
....
...
...
....
....
....
....
....
....
...
.
.
.
.
.
..
..
..
..
..
..
.
.
.
.
.
.
.
.
.
.
...
.
.
.
.
.
.
.
...................................................................................
...
.
.
.
.
.
.
...
.
.
.
.
.
.
..
..
..
..
..
..
.
.
.
.
.
...
.
.
.
.
.
...
.
...
...
...
...
...
...
..
..
...
..
..
..
.
.
.
.
.
.
.
.
.
.
..
.
...................................................................................................................................................................................................................................
.
.
.
.
.
.
..
.
.
.
.
.
...
.
.
.
.
.
.
.
...
...
...
...
...
...
..
...
.............................................................................................................................................................................................................

The figure (which illustrates


the case n = 6 and can easily be
generalized) shows that the 2n 2n
board can be blocked by 2n+1 n1
polyominoes (so that no more can
be accommodated).
To show that 2n+1 is minimal,
assume that the board is blocked by
2n polyominoes. Clearly, not all of
them can be of the same orientation
(horizontal or vertical).

We now state the evident fact that if, for any k , the k th line (row or column)
from an edge of the board contains a polyomino, then so also must the ith line
from the same edge for all i < k. Hence, the lines not containing a polyomino
must be consecutive if parallel.
Let there be p rows and q columns that do not contain a polyomino. Note
that p 1 and q 1. Then there is a p q rectangle on the board which is
uncovered. Since there are 2n polyominoes and 4n lines altogether, we must have
p + q 2n and either p n or q n. This shows that an additional polyomino
can be accommodated and provides the needed contradiction.
It is clear that the number of squares left vacant is maximum when the
number of polyominoes used is minimum. Hence, the maximum number of squares
left vacant is

(2n)2 n(2n + 1) = 2n2 n.

28

1456.

[1989 : 178]

1. Find a pair of integers (a, b) such that


x13 233x 144 and x15 + ax + b
have a common (non-constant) polynomial factor.
2.? Is the solution unique?
I. Solution to (a) by Mathew Englander, Kitchener, Ontario. [1990 : 250]
Consider the Fibonacci sequence f0 = f1 = 1, fn+2 = fn+1 + fn . For n 0
let pn be the polynomial
xn + xn1 + 2xn2 + 3xn3 + 5xn4 + + fn1 x + fn .
Then
(x2 x 1)pn

= xn+2 + xn+1 + 2xn + + fn1 x3 + fn x2

(xn+1 + xn + 2xn1 + + fn1 x2 + fn x)


(xn + xn1 + 2xn2 + + fn1 x + fn )

= xn+2 fn+1 x fn .
Observe that f11 = 144, f12 = 233, f13 = 377, f14 = 610. Thus,
(x2 x 1)p11 = x13 233x 144
and
(x2 x 1)p13 = x15 610x 377.
Thus, (a, b) = (610, 377) is one solution to the problem.

II. Solution to (b) by Stanley Rabinowitz, Westford, Massachusetts, USA.


[1990 : 251]
[Of course, Rabinowitz first answered part (a). Ed.]
My computer tells me that x13 233x 144 factors as

x2 x 1 x11 + x10 + 2x9 + 3x8 + 5x7 + 8x6


+13x5 + 21x4 + 34x3 + 55x2 + 89x + 144 , (1)

where both factors are irreducible. (This factorization, involving Fibonacci


numbers, can be found in Charles R. Wall, Problem B55, Fibonacci Quarterly
3 (1965) 158, and Russel Euler, Problem 8, Missouri Journal of Math. Sciences 1
(1989) 4445.) If x15 + ax + b has a (non-constant) polynomial factor in common
with x13 233x 144, then this common factor must be one of the two factors in
(1) (or their product).

29

Case (i).
Suppose that x15 + ax + b is divisible by x2 x 1. Then since
15
x 610x 377 is also divisible by x2 x 1, the difference between these two
fifteenth degree polynomials must also be divisible by x2 x 1. In other words,
(a + 610)x + (b + 377) would be divisible by x2 x 1. The only way this could
happen is if a = 610 and b = 377.
Case (ii).

Suppose that x15 + ax + b is divisible by

x11 + x10 + 2x9 + 3x8 + 5x7 + 8x6 + 13x5 + 21x4 + 34x3 + 55x2 + 89x + 144.
In this case we have
x15 + ax + b = (x4 + px3 + qx2 + rx + s)
or

x13 233x 144


,
x2 x 1

(x15 + ax + b)(x2 x 1) = (x4 + px3 + qx2 + rx + s)(x13 233x 144).


But this equation cannot hold, since the coefficient of x5 is 0 on the left and 233
on the right.
Thus, the solution in part (a) is unique.
Editors comment. [1990 : 252]
In addition Richard I. Hess, Rancho Palos Verdes, California, USA, and
Robert E. Shafer, Berkeley, California gave almostcomplete proofs of part (b).
Hesss dealt with the stronger problem of showing that no integers (a, b) 6=
(610, 377) exist such that x15 +ax+b has a root in common with x13 233x144.
Shafers solution was like Rabinowitzs but without his computer, and thus, was
much longer. Shafer also wonders about the irreducibility of the polynomial
xn Fn+1 x Fn
= xn2 + xn3 + 2xn4 + + Fn1 x + Fn ,
x2 x 1
where Fn is the nth Fibonacci number. A computer verifies that these are all
irreducible for n 13. Can anyone come up with further information?

2054.

[1995 : 202]
Are there any integral solutions of the Diophantine equation

(x + y + z)3 = 9 x2 y + y 2 z + z 2 x

other than (x, y, z) = (n, n, n)?

I. Solution by Adrian Chan, student, Upper Canada College, Toronto,


Ontario. [1996 : 188]
No, there are no integral solutions other than (x, y, z) = (n, n, n).
Without loss of generality, let x y and x z. Let y = x + a and z = x + b,
where a and b are non-negative integers. Then the given equation becomes

(3x + a + b)3 = 9 x2 (x + a) + (x + a)2 (x + b) + (x + b)2 x .

30

After expanding and simplifying, this is (a + b)3 = 9a2 b, or


a3 6a2 b + 3ab2 + b3 = 0.

(1)

Let a = kb, where k is a rational number. Then (1) becomes


b3 (k 3 6k 2 + 3k + 1) = 0.
By the Rational Roots Theorem, k 3 6k 2 + 3k + 1 = 0 does not have any rational
roots. Thus, since k is rational, k 3 6k 2 + 3k + 1 6= 0. Therefore, b = 0 and a = 0,
so that x = y = z.
II. Solution by Murray S. Klamkin, University of Alberta, Edmonton,
Alberta. [1996 : 188]
Letting y = x + u and z = x + v, the equation reduces to
(u + v)3 = 9u2 v,

(2)

where u and v are integers. We now show that the only solution to (2) is u = v = 0
so that (x, y, z) = (n, n, n) is the only solution of the given equation. Letting
u + v = w, (2) becomes
w3 = 9u2 (w u).
(3)

Hence, w = 3w1 where w1 is an integer, and (3) is 3w13 = u2 (3w1 u). It follows
that u = 3u1 for some integer u1 , and we get
w13 = 9u21 (w1 u1 ).
Comparing this equation to (3), we see by infinite descent that the only solution
to (3) is u = w = 0, which gives the negative result.

1863.

[1993 : 203]
Are there any integer solutions of the equation
(x + y + z)5 = 80xyz(x2 + y 2 + z 2 )

such that none of x, y, z are 0?


Solution by Murray S. Klamkin, University of Alberta, Edmonton, Alberta.
[1994 : 173]
No. The identity
(x + y + z)5 (x + y + z)5 (x y + z)5 (x + y z)5 = 80xyz(x2 + y 2 + z 2 )
follows by expanding out. Hence, the given equation is equivalent to
(x + y + z)5 + (x y + z)5 + (x + y z)5 = 0.

(1)

Since Fermats Last Theorem is known to be valid for exponent 5, there are only
the trivial solutions x = 0, y + z = 0 and symmetrically. [For if x + y + z = 0,
(1) becomes 05 + (2z)5 + (2y)5 = 0, which implies z = y, so that x = 0 = y + z.]

31

The mysterious procedure by which the proposer conjures up identities such


as the one above seems to be unavailable to the rest of us! Just two readers guessed
that the answer to the problem was no, and neither had a complete proof.
P. Penning, Delft, the Netherlands, investigated when the number 80 of the
problem can be replaced by a positive integer N so that the resulting equation has
an all non-zero integer solution. Via a computer search, he found only two such
values of N : N = 81, with solutions x = y = z; and N = 108, with solutions
4x = 4y = z. Any others?

1027.

[1985 : 83, 248]


Determine all quadruples (a, b, c, d) of non-zero integers satisfying the
Diophantine equation
abcd

1 1 1 1
+ + +
a b
c d

2

= (a + b + c + d)2

and such that a2 + b2 + c2 + d2 is a prime.


Solution par C. FestraetsHamoir, Bruxelles, Belgique. [1986 : 160]
2
1 1 1 1
+ + +
= (a + b + c + d)2
a b
c d


1
1
1
1
2
2
2
2
2
2
= abcd 2 + 2 + 2 + 2 +
+
+
+
+
+
a
b
c
d
ab ac ad bc bd cd

abcd

= a2 + b2 + c2 + d2 + 2ab + 2ac + 2ad + 2bc + 2bd + 2cd


bcd acd abd abc
=
+
+
+
= a2 + b2 + c2 + d2
a
b
c
d
= b2 c2 d2 + a2 c2 d2 + a2 b2 d2 + a2 b2 c2 = abcd(a2 + b2 + c2 + d2 ).
Posons a2 + b2 + c2 + d2 = p (p premier). On a
b2 c2 d2 + a2 c2 d2 + a2 b2 d2 + a2 b2 c2 0 (mod p)
b2 c2 (d2 + a2 ) + a2 d2 (c2 + b2 ) 0 (mod p)
b2 c2 (b2 c2 ) + a2 d2 (c2 + b2 ) 0 (mod p)
(a2 d2 b2 c2 )(b2 + c2 ) 0 (mod p)
a2 d2 b2 c2 0 (mod p) car (b2 + c2 ,p) = 1.
De m
eme, on a
a2 c2 b2 d2 0 (mod p).
Do`
u, par addition
a2 d2 b2 c2 + a2 c2 b2 d2 0 (mod p)
(a2 b2 )(d2 + c2 ) 0 (mod p)
a2 b2 0 (mod p) car (d2 + c2 ,p) = 1.

32

Par symetrie, om obtient


a2 b2 c2 d2 (mod p)
p = a2 + b2 + c2 + d2 4a2 (mod p),
a2 = 0 ou a2 = p
ce qui est impossible. Donc, il nexiste aucun quadruple dentiers positifs non nuls
(a,b,c,d) satisfaisant les conditions donnees.

969.

[1984 : 217]
Find a 3-parameter solution of the Diophantine equation
x
y
z
2w2
p
+
+
=
.
x2 + w 2
y 2 + w2
z 2 + w2
(x2 + w2 )(y 2 + w2 )(z 2 + w2 )

(1)

Solution by Murray S. Klamkin, University of Alberta, Edmonton, Alberta


(revised by the editor) [1985 : 300]
It is clear that (x, y, z, w) is a solution of (1) if and only if (kx, ky, kz, kw) is a
solution for any k > 0, so that it will suffice for our problem to find a 3parameter
family of primitive solutions, corresponding to k = 1.
We first show that (1) is satisfied by infinitely many solutions of
yz + zx + xy = w 2 ,

(2)

so that it will make sense to look for solutions of (1) among those of (2). It is
clear that (1) is satisfied by any solution of (2) in which w = 0 and xyz 6= 0. For
solutions of (2) with w 6= 0, we can choose x, y, z, such that
x
= cot A,
w

y
= cot B,
w

z
= cot C,
w

(3)

where A, B, C, are angles of a triangle. This can be done in infinitely many ways
for each w 6= 0. Substituting (3) into (2) and then into (1) gives
cot B cot C + cot C cot A + cot A cot B = 1
and
sin 2A + sin 2B + sin 2C = 4 sin A sin B sin C,
respectively, both of which are valid triangle identities. Hence, every solution (3)
satisfies (2) and (1).
To solve (2), we use the method of Desboves and assume that
x = a + p,

y = a + q,

z = r,

w = a + t,

where 6= 0, is a solution of (2). Substituting these values into (2) gives


=

a(2t p q 2r)
.
qr + rp + pq t2

(4)

33

Since (2) is homogeneous, we can multiply (4) by qr + rp + pq t2 and obtain


x
y
z
w

= a(qr + rp + pq t2 ) + ap(2t p q 2r),


= a(qr + rp + pq t2 ) + aq(2t p q 2r),
=

ar(2t p q 2r),

= a(qr + rp + pq t ) + at(2t p q 2r).

With m = t p and n = t q, we get


x = ar(m n) m2 ,
y = ar(n m) n2 ,

z
w

= ar(m + n 2r),
= amn r(m + n).

It will be found that these values satisfy (2) for all a, m, n, r, but they satisfy (1)
for all m, n, r, if and only if a < 0. [To facilitate the latter verification process,
we note that
x2 + w 2
y 2 + w2
z 2 + w2

= a2 (m2 + n2 )(m2 2rm + 2r2 ),


= a2 (m2 + n2 )(n2 2rn + 2r2 ),

= a2 (m2 2rm + 2r2 )(n2 2rn + 2r2 )].

Consequently, we take a = 1 and obtain the following 3parameter primitive


solution set of (1):
x = m2 r(m n),
y
z

= n2 r(n m),
= r(2r m n),

= r(m + n) mn.

Editors comment [1985 : 301]


Equation (2) is extensively discussed in L.J. Mordell, Diophantine Equations,
Academic Press, New York, 1969, pp. 291292.

1561.

[1990 : 204]
Determine an infinite class of integer triples (x, y, z) satisfying the Diophantine
equation
x2 + y 2 + z 2 = 2yz + 2zx + 2xy 3.
Solution by Hayo Ahlburg, Benidorm, Spain. [1991 : 252]
The well-publicized identity (see Leo Sauves footnote on [1976 : 176])
12 + (n2 n + 1)2 + (n2 + n + 1)2
= 2(n2 n + 1) + 2(n2 n + 1)(n2 + n + 1)
+2(n2 + n + 1) 3

(1)

34

is one answer to this problem if we choose n to be any integer. To find all solutions,
we rearrange the original equation and get
z 2 2(x + y)z + (x y)2 + 3 = 0
and
z = x+y

4xy 3.

To make z an integer, 4xy 3 must be the square of an odd number 2n + 1; that


is, 4n2 + 4n + 1 = 4xy 3 or
n2 + n + 1 = xy,
where n can be any integer. We can choose x and y as factors of n2 + n + 1 (this
can sometimes be done in several ways), and with
z = x + y (2n + 1)
this solves the original equation. Due to the symmetry of the problem, any
permutation of the values for x, y and z is also a solution. n, x, y and z can of
course also have negative values. Using both signs for x, y, z, and for 2n + 1 in the
expression for z leads to duplications. But using + signs throughout already gives
an infinite number of solutions. Factoring n2 + n + 1 into x = 1 and y = n2 + n + 1,
and with z = x + y (2n + 1), we get equation (1).
A nice special group is the series

1, 1, 3, 7, 19, 49, 129, 337, 883, 2311, 6051, . . .,


where each term has the form F2n+1 Fn Fn+1 made up of Fibonacci numbers.
Any three consecutive numbers of this series form a solution.

2034. [1995 : 130, 157] Murray S. Klamkin and M.V. Subbarao, University of
Alberta, Edmonton, Alberta.
(a) Find all sequences p1 < p2 < < pn of distinct prime numbers such that


 

1
1
1
1+
1+
1+
p1
p2
pn
is an integer.
(b) Can

1+

1
a21



1+

1
a22



1
1+ 2
an

be an integer, where a1 , a2 , . . . are distinct integers greater than 1?


Solution to (a), by Heinz-J
urgen Seiffert, Berlin, Germany. [1996 : 138]
If the considered product is an integer, then pn |(pi + 1) for some i {1, 2,
. . ., n 1}. Since pi + 1 pn , it then follows that pn = pi + 1, which implies
pi = 2 and pn = 3. Thus, n = 2, p1 = 2, p2 = 3. This is indeed a solution since
1 + 21 1 + 31 = 2.

35

Solution to (b). [1996 : 138]


In the following, we present four different solutions submitted by eight solvers
and the proposers. In all of them, p denotes the given product, and it is shown
that 1 < p < 2 and thus, p cannot be an integer. Clearly one may assume, without
loss of generality, that 1 < a1 < a2 < . . . < an .
Solution I, by Carl Bosley, student, Washburn Rural High School, Topeka, KS,
USA; Kee-Wai Lau, Hong Kong; and Kathleen E. Lewis, SUNY, Oswego, New
York. [1996 : 138]
Since 1 + x < ex for x > 0, we have


1
1
1
1 < p < exp
+ 2 +... + 2
a2
a2
a
 n
 1
1
1
+ 2 +...
< exp
22
3

 2

= exp
1 1.90586 < 2.
6
Solution II, by Toby Gee, student, the John of Gaunt School, Trowbridge,
England. [1996 : 138]
Since a1 2, we have
n+1
Y


n+1
n+1
Y k2 + 1
Y k2
1
1 < p
1+ 2 =
<
k
k2
k2 1
k=2
k=2
k=2
!
!
n+1
n+1
Y k
Y k
2(n + 1)
=
=
< 2.
k+1
k1
n+2
k=2

k=2

Solution III, by Walther Janous, Ursulinengymnasium, Innsbruck, Austria;


V
aclav Kone
ny
, Ferris State University, Big Rapids, MI, USA; Heinz-J
urgen
Seiffert, Berlin, Germany; and Murray S. Klamkin and M.V. Subbarao, University
of Alberta, Edmonton, Alberta. [1996 : 138]




Y
1
1Y
1
sinh
1<p<
1+ 2 =
1+ 2 =
1.838 < 2.
t
2
t
2
t=2
t=1

Solution IV, Richard I. Hess, Rancho Palos Verdes, California, USA.


[1996 : 138]


Y
1
Let p =
1 + 2 . Then clearly 1 < p < p = Q R where Q =
h
k=2



10

Y
Y
1
1
1 + 2 1.6714 and R =
1+ 2 .
h
k
k=2
k=11




Z
Z

X
1
1
dx
1
Now, ln R =
ln 1 + 2 <
ln 1 + 2 dx <
=
. (Ed: This
2
k
x
10
10
10 x
k=11

36


1
is because f (x) = ln 1 + 2 is strictly decreasing on (0, ) and ln(1 + t) < t
x
for t > 0).
Therefore, R < e0.1 1.1052, and thus, p < (1.68)(1.11) = 1.8648 < 2.

1752?.

[1992 : 175]
If A and B are positive integers and p is a prime such that p | A, p2 6 | A and
p2 | B, then the arithmetic progression
A, A + B, A + 2B, A + 3B, . . .
contains no terms which are perfect powers (squares, cubes, etc.). Are there any
infinite non-constant arithmetic progressions of positive integers, with no term a
perfect power, which are not of this form?
I. Solution by Margherita Barile, student, Universit
at Essen, Germany.
[1993 L 149]
The answer is in the affirmative. Let p be a prime, p > 2. Since
12 (p 1)2 1 (mod p), there is an r satisfying 0 < r < p, such that a2 6 r
(mod p) for all a = 0, . . . , p 1. Let A = p2 rp , B = p3 . Then, for all k, we have
A + kB = p2 rp + kp3 = p2 (rp + kp).
The greatest power of p dividing A + kB is p2 . Hence, if A + kB is a perfect power,
it is a square. Then there is an integer s such that
rp + kp = s2 .
But then, by Fermat,
s2 r p r

(mod p),

which provides a contradiction. Thus, the arithmetic progression A, A+B, A+2B,


. . ., contains no perfect powers. But it is not of the given form. In fact, our proof
shows how to construct infinitely many such arithmetic progressions. We give one
example. For p = 3, one gets r = 2; then A = 72, and B = 27.
II. Solution by Leroy F. Meyers, Ohio State University, Ohio, USA. [1993 : 150]
For each non-negative integer m let xm = A + mB.
The proof of the first statement of the problem is trivial. (In fact, it may not
have been intended to be part of the problem.) Suppose that p is a prime and
that p | A, p2 6 | A, and p2 | B. Then xm A 6 0 (mod p2 ), so that p | xm and
p2 6 | xm . But if xm is a perfect k th power, then pk must divide xm , which is
impossible if k > 1.
Two cases of the converse are considered.
If no prime dividing A occurs to a higher power in B than in A, then A is
relatively prime to C = B/ gcd(A, B). By Eulers theorem we have
A(C) 1 (mod C),

37

so that
A1+(C) A (mod AC).
but AC = AB/ gcd(A, B) = lcm(A, B), and thus, B | AC. Hence,
A1+(C) A (mod B),
and A1+(C) is the required perfect power in the arithmetic sequence.
However, if A is divisible by a prime which occurs to a higher power in B, then
there may be no perfect power in the sequence. For example, let A = 12 = 22 3
and B = 16 = 24 . If
12 + 16m = sk

for some positive integer k,

then s must be even, in which case 12 + 16m must be divisible by 2k , which is


impossible if k > 2. However, if k = 2 and s = 2t, then
3 + 4m = t2 ,
which also is impossible, since a perfect square must be congruent to either 0
or 1 modulo 4. Thus, there can be a non-constant perfect-power-free arithmetic
sequence not of the specified form.

1434. [1989 : 110] Proposed by Harvey Abbott and Murray S. Klamkin,


University of Alberta, Edmonton, Alberta.
It is known that
(3m)!(3n)!
,
m!n!(m + n)!(n + m)!

(4m)!(4n)!
,
m!n!(2m + n)!(2n + m)!

(5m)!(5n)!
m!n!(3m + n)!(3n + m)!
are all integers for positive integers m, n.
1. Find positive integers m, n such that
I(m, n) =

(6m)!(6n)!
m!n!(4m + n)!(4n + m)!

is not an integer.
2. Let A be the set of pairs (m, n), with n m, for which I(m, n) is not an
integer, and let A(x) be the number of pairs in A satisfying 1 n m x.
Show that A has positive lower density in the sense that
lim inf

A(x)
> 0.
x2

38

Solution by Marcin E. Kuczma, Warszawa, Poland. [1990 : 184]


The clue to (ii) is the observation that the set
Q = {(x, y) : 0 y x 1, b6xc+c6yc < b4x + yc + b4y + xc}

(1)

is non-empty ( here bxc is the greatest integer x); namely

Q = {(x, y) : 4x + y 2, 4y + x 1, x < 1/2, y > 1/6}

(2)

is the quadrangle with vertices



 
 
 

1 1
11 1
7 2
1 1
,
,
,
,
,
,
,
.
2 6
24 6
15 15
2 8
(There is no need to verify that the sets (1) and (2) are equal; it suffices for the
sequel to take (2) as the definition of Q.) What follows is routine. Choose a prime
p > 2 and suppose (m, n) is a lattice point in
pQ = {(px, py) : (x, y) Q}.
By (2)

6n
4m + n
4n + m
6m
< 3,
< 1,
2,
1.
p
p
p
p
Thus, p enters the denominator of I(m, n) with the exponent
 

    
n
4m + n
4n + m
m
+
+
+
3
p
p
p
p
n < m,

(3)

and numerator with the exponent



  
6m
6n
+
2
p
p

(there are no higher order terms, since p2 exceeds 6m). Consequently


[
(Z2 pQ) A.
p prime

The claim thus follows immediately: denoting by p(x) the greatest prime less than
or equal to x, we have asymptotically p(x) x and thus,


A(x)
A(p(x))
|Z p(x)Q|
1

Area
Q
=
x2
x2
x2
720
as x .
An example of (i) would be m = 11, n = 3, which satisfy (3) for p = 23.

Editors comment. [1990 : 185]


The proposers ask whether lim (A(x)/x2 ) exists.
x

The problem of showing that


(5m)!(5n)!
m!n!(3m + n)(3n + m)!
is integral occurred in the 1975 U.S.A. Mathematical Olympiad (and was suggested
by Klamkin).

39

Functions and Polynomials

299.

[1976 : 298]

If
F1
F2
F3

= (r2 + s2 2t2 )(x2 y 2 2xy) 2rs(x2 y 2 + 2xy)

+ 4rt(x2 + y 2 ),
= 2rs(x2 y 2 2xy) + (r2 s2 2t2 )(x2 y 2 + 2xy)

+ 4st(x2 + y 2 ),
= 2rt(x2 y 2 2xy) 2st(x2 y 2 + 2xy)
+ (r2 + s2 + 2t2 )(x2 + y 2 ),

show that F1 , F2 and F3 are functionally dependent and find their functional
relationship. Also, reduce the five-parameter representation of F1 , F2 , and F3 to
one of two parameters.
Solution de F.G.B. Maskell, Coll`ege Algonquin, Ottawa Ontario. [1978 : 170]
Posons
A = x(r t) + y(s + t),
B = x(s t) y(r + t).
On verifie, avec un peu de patience, que
(
F3 + F2 = 2A(A B),
F3 F1 = 2A(A + B),

F3 F2 = 2B(A + B),

F3 + F1 = 2B(A B).

(1)

On a donc
(F3 + F2 ) (F3 F2 ) + (F3 + F1 ) (F3 F1 ) = 0,
ce qui am`ene
F12 + F22 = 2F32 .

(2)

Cest la relation fonctionnelle recherchee.


On verra que les deux param`etres A et B suffisent pour representer les trois
fonctions donnees. En effet, de
2F3 F1 + F2 = 4A2 et 2F3 + F1 F2 = 4B 2 ,
on obtient
F1 + F2 = 2(A2 B 2 )

(3)

F3 = A 2 + B 2 .

(4)

et
Portons maintenant dans (2) les valeurs de F2 et F3 de (3) et (4); il resulte

F12 + 2 A2 B 2 F1 + A4 6A2 B 2 + B 4 = 0,

40

do`
u F1 = A2 2AB + B 2 . Ici il faut prendre
F1 = A2 2AB + B 2 ,
car lautre choix, avec la valeur de F2 qui decoule alors de (3), ne verifie aucune
des relations (1). La representation recherchee est donc
F1 = A2 2AB + B 2 , F2 = A2 2AB B 2 , F3 = A2 + B 2 .
Editors comment.[1978 : 171]
The proposer mentioned that the problem arose in applying the method of
Desboves in obtaining the general solution of the Diophantine equation F12 +
F22 = 2F32 from the knowledge of one particular solution. An analogous but
more complicated set of equations would arise if we started with the general
homogeneous quadratic Diophantine equation in n variables and one particular
solution.

254.

[1976 : 155]

1. If P (x) denotes a polynomial with integer coefficients such that


P (1000) = 1000,

P (2000) = 2000,

P (3000) = 4000,

prove that the zeros of P (x) cannot be integers.


2. Prove that there is no such polynomial if
P (1000) = 1000,

P (2000) = 2000,

P (3000) = 1000.

Solution by Leroy F. Meyers, Ohio State University, Columbus, Ohio, USA.


[1978 : 50]
A generalization yielding both parts is proved.
Let a, b, c, d, k be integers, with k 6= 0. Suppose P is a polynomial with
integral coefficients such that
P (d k) = ak,

P (d) = bk,

and

P (d + k) = ck.

If we define Q by Q(x) = P (x+d), then Q is a polynomial with integral coefficients


and
Q(k) = ak, Q(0) = bk, and Q(k) = ck.
Now the polynomial Q1 of lowest degree such that
Q1 (k) = ak,

Q1 (0) = bk,

and

Q1 (k) = ck

is given, by the Lagrange interpolation formula or otherwise, by


Q1 (x) =

(a 2b + c)x2 + (c a)kx + 2bk 2


.
2k

41

Hence, the polynomial Q Q1 has zeros at k, 0, k, and thus,


Q(x) Q1 (x) = (x + k)x(x k)R(x)
for some polynomial R, or
2kQ(x) (a 2b + c)x2 (c a)kx 2bk 2
=


x3 k 2 x S(x),

(1)

where S = 2kR. Now S, being the quotient of a polynomial with integral


coefficients by a polynomial with integral coefficients and leading coefficient 1,
must itself have integral coefficients. Equating the coefficients of x2 on the two
sides of (1) yields
2kq (a 2b + c) = k 2 s,
where q and s are the coefficients of x2 in Q(x) and x in S(x), respectively, and it
follows that
a 2b + c is divisible by k.

(2)

If c a, and hence, c + a, is even, then slightly more can be proved. Instead of


(1) we now have
kQ(x)

a 2b + c 2 c a
x
kx bk 2 =
2
2


x3 k 2 x S(x),

(1)

where now S = kR. Then, as before, S has integral coefficients and, after
comparing the coefficients of x2 , we obtain
kq

a 2b + c
= k 2 s,
2

and thus,
a 2b + c
is divisible by k.
2

(2)

Note that (2) and (2) are independent of d, which can therefore be any integer.
In part (a) we have a = 1, b = 2, c = 4, so that a 2b + c = 1. Hence, by (2),
k is a divisor of 1, that is, k = 1.

= 1. Since c a is
In part (b) we have a = 1, b = 2, c = 1, so that a2b+c
2
even, we can now use (2) to conclude that k is a divisor of 1, that is, k = 1.
Since k = 1000 in both parts of the proposal, each part yields a contradiction.
Hence, there is no polynomial with integral coefficients which satisfies either of the
given conditions, and thus, none of the zeros of P can be integers; also, all of them
must be integers. (For an analogous situation, see Problem 138 [1976 : 157]).

42

1423?.

[1989 : 73]
Given positive integers k, m, n, find a polynomial p(x) with real coefficients
such that
(x 1)n | (p(x))m xk .
What is the least possible degree of p (in terms of k, m, n)?
Solution by Robert P. Israel, University of British Columbia, Vancouver, BC.
[1990 : 145]
If p is a polynomial in x, pm xk is divisible by (x 1)n if and only if
pm = xk + O((x 1)n ) as x 1.
[Editors note: This means that pm = xk + r(x), where
|r(x)| M |x 1|n

(1)

for some constant M and for x sufficiently close to 1.] This is true if and only if
p = xk/m + O((x 1)n ).
[Editors note: Len Bos contributes the following elaboration for the editor, who
regrets not paying more attention as a student during analysis class. We have
"
#
1/m
r(x)
p = xk/m +
1+ k
1 xk/m .
(2)
x
Applying the Mean Value Theorem to the function f (t) = (1 + t)1/m , t > 1,
yields that
1
|(1 + t)1/m 1| = |t| (1 + c)1/m 1
m
for some c, |c| < |t|, so that with t = r(x)/xk we get


1/m


r(x)
|r(x)| 1


1 =
(1 + c)1/m 1
1+ k


x
xk
m
for some c between 0 and r(x)/xk . For x sufficiently close to 1 this means


1/m


r(x)


1 xk/m K|x 1|n
1+ k


x
for some constant K, by (1). Thus, from (2) we get the result.]

This means that p consists of the terms of the Taylor series of xk/m about
x = 1 up to order (x 1)n1 , plus any combination of higher powers of x 1.
That Taylor series is
1+

n1
X
j=1

k/m(k/m 1) (k/m j + 1)
(x 1)j .
j!

(3)

43

If k is not divisible by m, all coefficients of the Taylor series are non-zero, so


that the least degree of p is n1. If k is divisible by m, the coefficients for j > k/m
are zero, so that the least degree of p is min(n 1, k/m).
Editors comment. [1990 : 146]
It appears that the series (3) can also be written as

ni
n1 (1)
k(k m)(k 2m) (k (n 1)m) X
i
xi ;
n1
(n 1)!m
k

im
i=0
n1i

can any reader supply a proof?


II. Comment by Rex Westbrook, University of Calgary, Calgary, Alberta.
[1991 : 83]
This is in response to the editors request for a proof that the published answer

1+

n1
X k(k
m m
j=1

k
1) ( m
j + 1)
(x 1)j
j!

can be written as
k k
m(m



n1
k
1) ( m
n + 1) X (1)n1i n 1 i
x.
k
(n 1)!
i
m i
i=0

Set the former expression equal to fn (x) and the latter equal to gn (x). The
asked equality obviously holds if k/m = t n 1, where t is an integer, because
then both expressions reduce to xt . Otherwise, consider
k
d k/m
k
[x
fn (x)] = x( m +1)
dx
m

n1
X k ( k 1) ( k j + 1) 
k
k
m m
m
+
j(x 1)j1 xk/m (x 1)j x m 1
j!
m
j=1



n1
k
k
k
X
( 1) ( m j + 1)
k +1)
k
k
( m
j1
m m

+
(x 1)
jx (x 1)
= x
m
j!
m
j=1





n1
k
k
X
( m j + 1)
k +1)
k
k
( m
j
j1
m

= x
+

j (x 1) + j(x 1)
m
j!
m
j=1

n1
X k ( k j + 1)
k
k
m
m
= x( m +1) +
(x 1)j1
m
(j 1)!
j=1

n1
X k ( k j + 1)( k j)
j
m
m
m

(x 1)
j!
j=1
" k k
#
k
m ( m 1) ( m
(n 1))
k +1)
( m
n1
= x
(x 1)
(n 1)!

44

and
"
#

k
X (1)n1i n1
1) ( m
(n 1)) d n1
i
ik/m
x
k
(n 1)!
dx i=0
i
m
n1
k k
k
X n 1 
(

1)

(n

1))
k
m
(1)x( m +1)
= m m
(1)n1i xi
(n 1)!
i
i=0
" k k
#
k
(n 1))
m ( m 1) ( m
k
= x( m +1)
(x 1)n1 .
(n 1)!
d k/m
[x
gn (x)] =
dx

k k
(
m m

Therefore,
i
d h k/m
x
(gn (x) fn (x)) = 0,
dx
so that
xk/m (gn (x) fn (x)) = constant.
If either k/m is not an integer, or is an integer n, then since gn fn is a
polynomial (of degree at most n 1) the above constant must be 0; that is,
gn f n .

2014.

[1995 : 52]

(a) Show that the polynomial




2 x7 + y 7 + z 7 7xyz x4 + y 4 + z 4

has x + y + z as a factor.

(b)? Is the remaining factor irreducible (over the complex numbers)?


I. Solution to (a) by Jayabrata Das, Calcutta, India. [1996 : 45]


Let f (x, y, z) = 2 x7 + y 7 + z 7 7xyz x4 + y 4 + z 4 . If we can show that
f (x, y, z) = z when x + y + z = 0, we are done.
We know, for x + y + z = 0, that x3 + y 3 + z 3 = 3xyz. Thus,

so that

x7 + y 7 + z 7 + x 3 y 4 + x 3 z 4 + y 3 z 4 + y 3 x4 + z 3 y 4 + z 3 x4


= x3 + y 3 + z 3 x4 + y 4 + z 4

= 3xyz x4 + y 4 + z 4

x7 + y 7 + z 7

= 3xyz x4 + y 4 + z 4

x3 y 4 x3 z 4 y 3 z 4 y 3 x4 z 3 y 4 z 3 x4

45

Therefore,


f (x, y, z) = 2 x7 + y 7 + z 7 7xyz x4 + y 4 + z 4

= 6xyz x4 + y 4 + z 4

2 x3 y 4 + x3 z 4 + y 3 z 4 + y 3 x4 + z 3 y 4 + z 3 x4

7xyz x4 + y 4 + z 4

= xyz x4 + y 4 + z 4

2x3 y 3 (x + y) 2y 3 z 3 (y + z) 2z 3 x3 (z + x)

= xyz x4 + y 4 + z 4 + 2x3 y 3 z + 2xy 3 z 2 + 2x3 yz 3

= xyz x4 + y 4 + z 4 2x2 y 2 2y 2 z 2 2z 2 x2

2

= xyz x2 + y 2 + z 2 4 x2 y 2 + y 2 z 2 + z 2 x2 .

Since x2 + y 2 + z 2 = 2(xy + yz + zx), we now have that



2
f (x, y, z) = xyz 4 (xy + yz + zx) 4 x2 y 2 + y 2 z 2 + z 2 x2

= 4xyz 2x2 yz + 2xy 2 z + 2xyz 2


= 8xyz xyz(x + y + z) = 0.

II. Solution to (a) by Cyrus Hsia, student, University of Toronto, Toronto,


Ontario. [1996 : 45]
Consider the sequence an = xn + y n + z n . The characteristic equation with
roots x, y, z, is
a3 Aa2 + Ba C = 0,
where A = x + y + z, B = xy + yz + zx and C = xyz.
The sequence {an } follows the recurrence relation:
an+3 = A an+2 B an+1 + C an .
Now, we have
a0

= x0 + y 0 + z 0 = 3,

a1
a2

= x1 + y 1 + z 1 = A,
= x2 + y 2 + z 2 = (x + y + z)2 2(xy + yz + zx) = A2 2B.

From the recurrence relation, we see:


a3

= A a2 B a1 + C a0
= A3 2AB AB + 3C
= A k3 + 3C,

where k3 is some term in x, y and z

46

Similarly
a4
a5
a6
a7

= A k4 + 2B 2 , where k4 is some term in x, y and z,


= A k5 5BC, where k5 is some term in x, y and z,

= A k6 2B 3 + 3C 2 , where k6 is some term in x, y and z,


= A k7 + 7B 2 C, where k7 is some term in x, y and z.

Thus,


2 x7 + y 7 + z 7 7xyz x4 + y 4 + z 4

= 2a7 7C a4


= a A k7 + 7B 2 C 7C A k4 + 2B 2
= A k,

where k is some term in x, y and z; that is,



x + y + z divides 2 x7 + y 7 + z 7 7xyz x4 + y 4 + z 4 .

1110?.

[1986 : 13]
How many different polynomials P (x1 , x2 , . . . , xm ) of degree n are there for
which the coefficients of all the terms are 0s or 1s and
P (x1 , x2 , . . . , xm ) = 1 whenever x1 + x2 + + xm = 1 ?

Editors comment. [1987 : 170]


There has been only one response to this problem, and it was incorrect. It
did note the easy case m = 1, where P (x) = xn is the only solution for each
n. Equally easy is the case n = 1 which has unique solution P (x1 , x2 , . . . , xm ) =
x1 + x2 + + xm for each m. I would not like to see this problem abandoned at
this point. Can anyone find all such polynomials for any other values of m and/or
n? Can anyone show that there is at least one such polynomial for each m and n?
I. Partial solution by Len Bos and Bill Sands, University of Calgary, Calgary,
Alberta. [1988 : 13]
Let f (n, m) be the required number of polynomials. We will investigate the
case m = 2 and will show that
 
1
2n
f (n, 2)
,
n+1 n
the nCatalan number.
Let P (x, y) be a polynomial of degree n with the required properties. Then
P (x, y) = 1 whenever x + y = 1, so that it must be true that
P (x, y) = (x + y 1) q(x, y) + 1

(1)

for some polynomial q(x, y) with integer coefficients. We shall count all those
possible q(x, y) whose coefficients are also all 0 or 1.

47

Let q be such a polynomial. Then its terms are monomials of the form xi y j ,
where i, j {0, 1, . . ., n 1} and i + j n 1. We will identify the collection of
these monomials with the corresponding subset of lattice points
Rq = {(i, j)|xi y j is a monomial in q}.
Thus, Rq is a subset of {(i, j) Z 2 |0 i, 0 j, i + j n 1} and contains
at least one (i, j) with i + j = n 1. The next two lemmas establish important
properties of Rq .
Lemma 1. If (i, j) Rq where i + j > 0, then either (i 1, j) Rq or
(i, j 1) Rq (or both). In particular, if (i, 0) Rq then (i 1, 0) Rq for i > 0,
and similarly for (0, j).
Proof. If (i, j) Rq and i + j > 0 then xi y j is a monomial in q. Thus,
(x + y 1)q(x, y), when multiplied out, will contain a term xi y j .
By (1), it must therefore also contain at least one term +xi y j , which can only
happen if xi1 y j or xi y j1 were monomials in q; that is, if (i1, j) or (i, j1) Rq .
In terms of lattice points, this lemma says that if a lattice point is in Rq , then
at least one of its neighbours to the left of or below it must also be in Rq .
Our other lemma is a sort of converse.
q

(i q1, j)

(i,qj)

(i, j 1)

Lemma 2. If (i 1, j) Rq and (i, j 1) Rq , then (i, j) Rq .


Proof. We have that xi1 y j and xi y j1 are both monomials in q. Then (x +
y 1)q(x, y) when multiplied out will contain two terms xi y j . By (1), it must also
contain a term xi y j , which implies that (i, j) Rq .
Now suppose (i, j) Rq . By applying Lemma 1 repeatedly, we obtain a
descending path of lattice points in Rq from (i, j) to (0, 0). By always moving left
from a lattice point rather than down, whenever we have a choice, we obtain what
we call the left path of (i, j). Similarly by moving down instead of left whenever
possible, we obtain the right path of (i, j). The diagram shows a possible left path
of (4, 3). All lattice points on the path are in Rq , but the position of the path tells

48

us that (2, 3) and (2, 2) are not in Rq .


q
q

..................................................
...
..
..
....
...
...
...
...
...
(2, 2)
...
...
...
...
.
.....................................................................................................................................................
....
..
.....
..
...
...
.

(2, 3)

(0, 0)

(4, 3)

Clearly the left path and right path of (i, j) do not cross, although they may
meet (and do, at their end-points at least).
q

(i, j)

...
...
..
.
..........................
.
...
...
...
...
....
...
....
...
..
....
.................................................
..
.
..
...
.
...
....
...
................................................
...
...
....
....
..
..
.......................................................................
...
...
...
...
...
..
..
.......................................................................

(0, 0)

(k, j)

(k, `)

Let (i, j) and (k, `) be in Rq , where we assume that i < k and j < `. Consider
the left path of (i, j) and the right path of (k, `). Extend them to paths beginning
at (k, j) by adding horizontal and vertical edges, respectively. The extended paths
then enclose a region of lattice points.
Claim:

Every lattice point inside this region is in Rq .

This follows by repeated applications of Lemma 2, starting at the bottom left


of the region and working up and to the right.
Now it can easily be seen that Rq must coincide with the region of lattice points
bounded by the left and right paths of some lattice point (i, j), where i+j = n1.
Furthermore we claim than any such region corresponds to a polynomial q(x, y)
such that P (x, y), defined by (1), is a polynomial satisfying the problem. We need
only show that P (x, y) has coefficients 0 or 1. Multiplying out (x + y 1)q(x, y),
we need only show that any negative term xi y j , i + j > 0, is offset by at least
one term +xi y j , and that if two terms xi y j occur then also a term xi y j will
occur. But this follows from the construction of the region much as in the proofs
of Lemmas 1 and 2.
q q q ...q
...
q q q ........q
..
...

q ....q..............P
......q..`
.....................q
...
....
.
q .......q q ........q Pr
...
...
..
..
..q.....................q....................q.....................q
(0, 0)
q q q q
(i, j)

(0, 0)

q q q ...q q
...
q q q ........q ....q
..
..
...
...
...q
q ....q..............P
.......q.`
...................q
....
.
....
.
.
q .....q q q ........q
...
...
..
q q .......q Pr
..q....................q
.
..
q ..q.....................q....................q........................q
(i, j)

49

Thus, to count all the polynomials q(x, y) we must count the number of pairs
of lattice paths P` and Pr from (0, 0) to (i, j), i + j = n 1, which do not cross
and have length n 1. By moving path Pr one unit to the right and one unit
down, adding in new common end-points as shown, and moving both paths to
start at (0, 0) again, we see that such pairs of paths correspond to those pairs of
paths from (0, 0) to (i, j), where i + j = n + 1, which do not meet (except at their
end-points), and which have length n + 1. The number of such pairs of paths, over
all choices of i, j satisfying i 0, j 0, i + j = n + 1, is known to be the Catalan
number
 
1
2n
n+1 n
(see J. Levine, Note on the number of pairs of non-intersecting routes, Scripta
Mathematica 24 (1959) 335338). This number is then a lower bound for f (n, 2).
Unfortunately it is not the exact answer, since putting
q(x, y) = 1 + x + y + 2xy + x2 y + xy 2 + x2 y 2
(which has a coefficient not equal to 0 or 1) into (1) yields
P (x, y) = x2 + y 2 + x2 y + xy 2 + x3 y + xy 3 + x2 y 2 + x3 y 2 + x2 y 3 ,
a polynomial with all coefficients 0 or 1. We do believe, however, that f (n, 2) can
be calculated, and, as a possible first step, make the following conjecture:
any q(x, y) suitable for (1) has all coefficients 0, 1 or 2.
II. Partial solution by P. Penning, Delft, the Netherlands. [1988 : 16] (Adapted
by the editor to refer to I above.)
We show that
f (n, m) mn1 ,
thus, answering the editors request [1987 : 170] for a proof that f (n, m) 1 for
each n and m.
A special case of the allowable regions in part I is that of a single path from
(0, 0) to (i, j), where i + j = n 1. A similar argument to that in I shows more
generally that P
if P is a path of length n 1 from (0, 0, . . . , 0) to (i1 , i2 , . . . , im )
m
in Z m , where j=1 ij = n 1, then the lattice points on P will correspond to
monomials whose sum is a polynomial q(x1 , . . . , xm ) such that
P (x1 , . . . , xm ) = (x1 + . . . + xm 1) q(x1 , . . . , xm ) + 1
has all coefficients 0 or 1. To construct such a path, we merely choose a sequence
of n 1 elements from x1 , . . ., xm , repetition allowed, each corresponding to one
of the m directions the path can take (starting at (0, . . . , 0)). The number of
these sequences is mn1 .
Example: m = 5, n = 4. Choose sequence x2 , x4 , x3 . Then
q(x1 , x2 , x3 , x4 , x5 ) = 1 + x2 + x2 x4 + x2 x4 x3 ,

50

so that
P (x1 , . . . , x5 ) = (1 + x2 + x2 x4 + x2 x4 x3 )(x1 + x2
+x3 + x4 + x5 1) + 1
= x1 + x3 + x4 + x5 + x2 (x1 + x2 + x3 + x5 )
+x2 x4 (x1 + x2 + x4 + x5 )
+x2 x3 x4 (x1 + x2 + x3 + x4 + x5 ).

1283.

[1987 : 289]
Show that the polynomial

P (x, y, z) = (x2 + y 2 + z 2 )3 (x3 + y 3 + z 3 )2

(x2 y + y 2 z + z 2 x)2 (xy 2 + yz 2 + zx2 )2

is non-negative for all real x, y, z.


Solution by Jorg Herterich, student, Winnenden, Federal Republic of Germany.
[1988 : 306]
This can be seen by writing the expression another way:
(x2 + y 2 + z 2 )3 (x3 + y 3 + z 3 )2
(x2 y + y 2 z + z 2 x)2 (xy 2 + yz 2 + zx2 )2

= 2x4 y 2 + 2x4 z 2 + 2y 4 x2 + 2y 4 z 2 + 2z 4 x2 + 2z 4 y 2
+ 6x2 y 2 z 2 2x3 y 3 2y 3 z 3 2x3 z 3 2x3 z 2 y

2x3 y 2 z 2y 3 x2 z 2y 3 z 2 x 2z 3 x2 y 2z 3 y 2 x
= (x2 y 2 + y 2 z 2 + x2 z 2 )(2x2 + 2y 2 + 2z 2 2xy 2yz 2xz)

= (x2 y 2 + y 2 z 2 + x2 z 2 )((x y)2 + (y z)2 + (x z)2 ).


It is obvious that this is non-negative for all real x, y, z.

51

Expressions and Identities

1304.

[1988 : 12]
If p, q, r are the real roots of
x3 6x2 + 3x + 1 = 0,

determine the possible values of


p2 q + q 2 r + r2 p,
and write them in a simple form.
Solution by Sam Baethge, Science Academy, Austin, Texas, USA. [1989 : 30]
Let
A = p2 q + q 2 r + r2 p,
B = p2 r + q 2 p + r2 q,
the only two possible values of expressions of the given type. We also have
p + q + r = 6,
pq + qr + rp = 3,
pqr = 1.
In the equations that follow, all summations are symmetric over p, q and r.
X
X
18 = (p + q + r)(pq + qr + rp) =
p2 q + 3pqr =
p2 q 3
or

A+B =

216 = (p + q + r)3 =
or

X
X
AB

p3 q 3 = 27 6(1) 3pqr

p2 q = 21.
p3 + 3

p2 q + 6pqr

p3 = 216 3(21) 6(1) = 159.

27 = (pq + qr + rp)3 =
or

(1)

p4 qr +

p3 q 3 + 3

p2 q = 21 3(1)(21) = 84.

p3 q 3 + 3p2 q 2 r2 = pqr

= (1)(159) + 87 = 72.
Using (1) and (2), A and B are the roots of
y 2 21y 72 = 0,
so that the possible values are 24 and 3.

p3 q 2 r + 6p2 q 2 r2

p3 + 84 + 3

(2)

52

287.

[1976 : 251]
Determine a real value of x satisfying
p
p
p
2ab + 2ax + 2bx a2 b2 x2 =
ax a2 + bx b2

if x > a, b > 0.

Composite of the solutions received from Gali Salvatore, Perkins, Quebec; and
Murray S. Klamkin, University of Alberta, Edmonton, Alberta. [1978 : 135]
The restriction 0 < a, b < x is equivalent to

b < x.
(1)
0 < a,
The radicand on the left of the given equation can be factored by inspection, giving
r

a+

 

 

 


b+ x
b+ x a
x+ a b
a+ b x

ax a2 +

bx b2 .

(2)

Now (1) ensures that the right side of (2) and the first three factors of the radicand
on the left are all positive; hence, for a solution to exist the fourth factor must
also be positive, and we require

x < a + b.
(3)
Thus, with (1) and (3) the given equation is equivalent to the one obtained by
squaring both sides. This procedure gives, after simplification,
np
o
x(a + b x) = 2 ab
(x a)(x b) ab

2x ab(x a b)
.
= p
(4)
(x a)(x b) + ab
One solution of (4) is clearly x = a + b; and it is the only one since x 6= a + b
makes one side of (4) positive and the other side negative.
Since x = a + b also satisfies both (1) and (3), it is the unique solution to the
given equation.
Editors comment.[1978 : 136]

A ....................................................x
M
x
......................................................................................................... C

...
..
.....
...
..
.....
...
.....
..
....
.....
.
..
.
...
.
.
.
..
.....
.....
......
....
.....
2 a .........
x ....
..... 2 b
....
...
.
.
.
.
.
.
...
..
..
...
.....
..
...
....
... .... .........
.... ... .....
...........
....

The restrictions
(1) and (3) ensure
that a, b, x (or any constant
multiple thereof) are the lengths of
the sides of a triangle. In fact,
the proposer pointed out that the
given equation can be interpreted
geometrically by the following area
relationship (see figure):

53

|ABC| = |ABM | + |M BC|.


This is most easily verified from (2) by Herons formula.
Thus, the given equation and its solution could be used to give us another
proof, if one were needed, of the following theorem:
If the mid-point of longest side of a triangle is equidistant from the three
vertices, then the triangle is rightangled.
Conversely, this theorem itself can be used to provide an unexpected solution
to the given equation.

1594.

[1990 : 298]
Express
x41 + x42 + x43 + x44 4x1 x2 x3 x4

as a sum of squares of rational functions with real coefficients. (By the AMGM
Inequality, this polynomial is non-negative for all real values of its variables, and
therefore, by a theorem of Hilbert, it can be so expressed.)
Solution by Iliya Bluskov, Technical University, Gabrovo, Bulgaria. [1992 : 22]
x41 + x42 + x43 + x44 4x1 x2 x3 x4
= x41 + x42 2x21 x22 + x43 + x44 2x23 x24 + 2x21 x22 + 2x23 x24 4x1 x2 x3 x4
2

2(x1 x2 x3 x4 ) .
= (x21 x22 )2 + (x23 x24 )2 +
Editors note.
It is known more generally that, for any even n,
an1 + an2 + + ann na1 a2 an
can be expressed as a sum of squares; see p. 55, section 2.23 of Hardy, Littlewood,
and P
olya, Inequalities, Cambridge Univ. Press.

1522.

[1990 : 74]
Show that if a, b, c, d, x, y > 0 and
xy = ac + bd,
then

x
ad + bc
=
,
y
ab + cd

abx
cdx
ady
bcy
+
=
+
.
a+b+x c+d+x
a+d+y b+c+y

I. Solution by Seung-Jin Bang, Seoul, Republic of Korea. [1991 : 127]


Note that, from the given expressions for xy and x/y,
ab(c + d + x) + cd(a + b + x) = ad(b + c + y) + bc(a + d + y)

54

and
x(a + d + y)(b + c + y) = ((a + d)x + ac + bd)(b + c + y)
= (ac + bd)y + (a + d)(b + c)x
+(b + c)(ac + bd) + (a + d)(ac + bd)
= (ac + bd)x + (a + b)(c + d)y
+(c + d)(ac + bd) + (a + b)(ac + bd)
= ((a + b)y + ac + bd)(c + d + x)
= y(a + b + x)(c + d + x).
It follows that

x

ab
cd
+
a+b+x c+d+x

= y

ad
bc
+
a+d+y b+c+y

II. Combination of partial solutions by Wilson da Costa Areias, Rio de Janeiro,


Brazil, and Francisco Bellot Rosado, I.B. Emilio Ferrari, Valladolid, Spain.
[1991 : 127]
It is well known (see p. 111, no. 207 of N. Altshiller-Court, College Geometry)
that it is possible to construct a cyclic quadrilateral ABCD of sides AB = a,
BC = b, CD = c, DA = d [provided that a < b + c + d, etc. Ed.] and also
that, from Ptolemys Theorem, its diagonals x = AC, y = BD satisfy
xy = ac + bd,

ad + bc
x
=
.
y
ab + cd

Let R be the circumradius of ABCD, and let F1 , F2 , F3 , F4 and s1 , s2 , s3 ,


s4 be the areas and semiperimeters of the triangles ABC, BCD, CDA, DAB,
respectively. Then
cdx
bcy
ady
abx
+
=
+
a+b+x c+d+x
b+c+y a+d+y
is equivalent to
F1 4R F3 4R
F2 4R F4 4R
+
=
+
,
2s1
2s3
2s2
2s4
or
r1 + r 3 =

F1
F3
F2
F4
+
=
+
= r2 + r4 ,
s1
s3
s2
s4

where r1 , r2 , r3 , r4 are, respectively, the inradii of the above triangles. The relation
r1 + r3 = r2 + r4 is true and has been shown by H. Forder in An ancient Chinese
theorem, Math Note 2128, p. 68 of Mathematical Gazette 34, no. 307 (1950). It
was also part (b) of Crux 1226 [1988 : 147].

55

830.

[1983 : 80]
Determine all real such that

|z1 z1 (z3 z2 ) + z2 z2 (z1 z3 ) + z3 z3 (z2 z1 ) iz1 |


= |(z2 z3 )(z3 z1 )(z1 z2 )| ,
where z1 , z2 , z3 are given distinct complex numbers and z1 6= 0.

Solution by Murray S. Klamkin, University of Alberta, Edmonton, Alberta


(revised by the editor). [1984 : 202]
Consider the triangle (assumed to be non-degenerate) whose affixes in the
complex plane are z1 , z2 , z3 , oriented in the sense of increasing subindices, and
let , R, denote its signed area, circumradius, and affix of its circumcentre,
respectively. It is known that
=

z1 z 1 (z3 z2 ) + z2 z 2 (z1 z3 ) + z3 z 3 (z2 z1 )


4i

and
R =

|(z2 z3 )(z3 z1 )(z1 z2 )|


.
4||

The given equation is therefore, equivalent to






+ z1 = R.

4

(1)

It is clear that (1) is satisfied if and only if

z1
= z
4

(2)

for some point z on the circumcircle, and then that is real if and only if z = z1
or z = z4 where z4 is the point where the line through the origin and z1 meets the
circumcircle again. For z = z1 , we obtain from (2)
= 1 = 4.
To obtain the value of when z = z4 in (2), we first note that the power of the
origin with respect to the circumcircle is given, in both magnitude and sign, by
the real number z 1 z4 = ||2 R2 . Hence, z = z4 in (2) when

4 ||2 R2
= 2 =
.
|z1 |2
Editors comment [1984 : 203]
Henderson found the values of 1 and 2 explicitly in terms of z1 , z2 , z3 :
1 = i

z 1 (z2 z3 ),

where the sums are cyclic.

2 =

i
|z1 |2

z 1 z 2 z3 (z2 z1 ),

56

1996.

[1994 : 285]
(a) Find positive integers a1 , a2 , a3 , a4 such that
(1 + a1 )(1 + a2 )(1 + a3 )(1 + a4 )
is an integer, where is a complex cube root of unity.
(b)? Are there positive integers a1 , a2 , a3 , a4 , a5 , a6 so that
(1 + a1 )(1 + a2 )(1 + a3 )(1 + a4 )(1 + a5 )(1 + a6 )
is an integer, where is a complex fifth root of unity?
I. Solution by Kee-Wai Lau, Hong Kong. [1995 : 311]

(a) We have(1 + 2)4 = 9. [Editors note. Since = 1/2 ( 3/2)i, we have


1 + 2 = 3 i.]
(b) We shall answer in the negative. We first show that if A, B, C, D are integers
such that
A + B + C 2 + D 3 = 0,
(1)

then A = B = C = D = 0. We may assume that = e2i/5 = cos 72 + i sin 72 ,


as the proofs for other choices of are similar. The real part and imaginary part
of the left hand side of (1) both vanish and thus,

and

A + B cos 72 (C + D) cos 36 = 0

(2)

B sin 72 + (C D) sin 36 = 0.

(3)

From (3) we obtain


2B cos 36 + (C D) = 0. Since cos 36 is irrational [in

fact, cos 36 = ( 5 + 1)/4 Ed.], B = 0 and C = D. Thus, (2) becomes


A 2C cos 36 = 0 and thus, A = C = 0. [Editors note. Alternatively, this result
is true because the polynomial 1 + z + z 2 + z 3 + z 4 is irreducible and thus, must be
the polynomial of smallest degree (with integer coefficients) that has as a root.
See the (first) Editors Note in Solution III below.]

Now suppose, on the contrary, that the answer is in the affirmative. Using the
fact that 4 = (1 + + 2 + 3 ) and 5 = 1, we see that the given product
equals P + Q + R 2 + T 3 , where P , Q, R, T are integers and R = S2 S4 ,
T = S3 S4 where Sk stands for the k th elementary symmetric function of a1 , a2 ,
. . ., a6 , k = 1, 2, 3, 4. Using the above result we see that R = T = 0 and thus,
S2 S3 = 0. However, since ak 1 we have
a1 a2 a 1 a2 a3 ,

a1 a3 a 1 a3 a4 ,

a1 a4 a 1 a4 a5 ,

a1 a5 a 1 a5 a6 ,

a1 a6 a 1 a2 a6 ,

a2 a3 a 2 a3 a4 ,

a2 a4 a 2 a4 a5 ,

a2 a5 a 2 a5 a6 ,

a2 a6 a 2 a3 a6 ,

a3 a4 a 3 a4 a5 ,

a3 a5 a 2 a3 a5 ,

a3 a6 a 3 a4 a6 ,

a4 a5 a 4 a5 a6 ,

a4 a6 a 2 a4 a6 ,

a5 a6 a 3 a5 a6 .

57

[Editors note. If the editor may put on his combinatorial hat for a minute,
what is going on here is just a demonstration of the known fact that there is a
complete matching from the 2element subsets to the 3element subsets of the
set {1, 2, 3, 4, 5, 6}; that is, a one-to-one function from the 2element subsets to
the 3element subsets such that each 2element subset is mapped to a 3element
subset containing it. In fact there is a complete matching from the kelement
subsets of an nelement set to the (k + 1)element subsets whenever k < n/2. See,
for example, Corollary 13.3 on page 688 of [2], or (for the whole story and more)
Chapters 1 to 3, especially Exercise 2.4 on page 23, of [1].] Therefore,
S2 S3 (a1 a2 a4 + a1 a2 a5 + a1 a3 a5 + a1 a3 a6 + a1 a4 a6 ) < 0,
a contradiction.
References:
[1] Ian Anderson, Combinatorics of Finite Sets, Oxford University Press, 1987.
[2] Ralph P. Grimaldi, Discrete and Combinatorial Mathematics (3rd Edition),
AddisonWesley, 1994.
II. Solution to part (a) by Shawn Godin, St. Joseph Scollard Hall, North Bay,
Ontario. [1995 : 311]
We show that the only solutions (up to permutations) are
(a1 , a2 , a3 , a4 ) = (1, 2, 3, 5) and (2, 2, 2, 2).
The given product equals A + B, where
A = 1 + (a1 a2 a3 + a1 a2 a4 + a1 a3 a4 + a2 a3 a4 ) 3
= 1 + (a1 a2 a3 + a1 a2 a4 + a1 a3 a4 + a2 a3 a4 ),
which is an integer, and
B

= (a1 + a2 + a3 + a4 )
+(a1 a2 + a1 a3 + a1 a4 + a2 a3 + a2 a4 + a3 a4 ) 2 + a1 a2 a3 a4 4
= (a1 + a2 + a3 + a4 )
+(a1 a2 + a1 a3 + a1 a4 + a2 a3 + a2 a4 + a3 a4 ) + a1 a2 a3 a4 .

Now to get rid of any imaginary parts the coefficients of and must be the
same. (And note that since the real parts of and are 1/2, if these coefficients
are equal, B, and thus, the given product, will be an integer.) Thus, we need
a1 + a 2 + a 3 + a 4 + a 1 a2 a3 a4
= a 1 a2 + a 1 a3 + a 1 a4 + a 2 a3 + a 2 a4 + a 3 a4 .
Without loss of generality assume 1 a1 a2 a3 a4 . If a1 a2 6, then

(4)

58

a1 + a 2 + a 3 + a 4 + a 1 a2 a3 a4
> 6a3 a4 a1 a2 + a1 a3 + a1 a4 + a2 a3 + a2 a4 + a3 a4 ;

so that a1 a2 < 6, and we have two cases: a1 = 1 and a1 = 2.


Case 1: a1 = 1. Then (4) reduces to
1 + a 2 a3 a4 = a 2 a3 + a 2 a4 + a 3 a4 .

(5)

If a2 3 then
1 + a2 a3 a4 > 3a3 a4 a2 a3 + a2 a4 + a3 a4 ;
so that a2 = 1 or 2. If a2 = 1, (5) reduces to 1 = a3 + a4 which has no positive
solution. If a2 = 2, (5) reduces to 1 + a3 a4 = 2(a3 + a4 ) which has solution a3 = 3,
a4 = 5. [Editors note. We borrow from Janouss solution and write this equation
as
(a3 2)(a4 2) = 3,
which makes it clear that a3 = 3, a4 = 5 is the only possibility.]

Case 2: a1 = 2. Since a1 a2 < 6, a2 = 2 also, and (4) reduces to a3 a4 = a3 + a4 ,


which has a3 = 2, a4 = 2 as its only positive solution.
III. Generalization of part (b) by Gerd Baron, Technische Universit
at Wien,
Austria. [1995 : 311]
We will consider the following more general situation: for n, m positiveQintegers
n
and m = 1, determine all sets {a1 , . . ., an } of positive integers such that i=1 (1+
ai ) is an integer. We will prove:
THEOREM. If m > 3 is prime and is a complex mth root of unity, then,
for
Qn m n 2m 3 and a1 , a2 , . . ., an non-negative integers, the product
i=1 (1 + ai ) is an integer only if some ai = 0.
Note that for m = 5 and 5 n 7, we get no solutions and (b) is done.

Proof of the theorem. Assume all ai > 0. As a polynomial in ,


n
Y

(1 + ai ) =

i=1

n
X

pk k =: P ()

k=0

where the pk are the elementary symmetric functions in ai , with


X
Y
p0 = 1, p1 =
ai , and pn =
ai .

Let n 2m 3. Reducing P () modulo the relation m = 1 we get


Q() =

m1
X

k=nm+1

pk k +

nm
X

(pk + pk+m ) k .

k=0

If m is prime, then the polynomial R() = m1 + m2 + + +1 is irreducible,


and Q() = u is an integer exactly if the polynomial Q()u is an integer multiple
of R(); that is, all coefficients of Q() but the constant term are equal.

59

[Editors note. It is known that if m is prime, the polynomial xm1 + xm2 +


+ x + 1 is irreducible; that is, it cannotbe factored into polynomials of smaller
degree with integer coefficients. For example, see exercise 19, page 84 of Ed
Barbeaus Polynomials (SpringerVerlag, 1989), or most any text on abstract
algebra. Now since is a root of the polynomials R() and Q() u, it will
be a root of their greatest common divisor, which must be R() since R() is
irreducible; thus, Q() u must be a multiple of R(), and a constant multiple
since they both have degree m 1.]
If we can show that pk 6= pk+1 for some k with n m < k < m 1, we are done
and there is no solution with all ai > 0. We claim that if m is a prime greater than
3, and m n 2m 3, then there is such a k with pk+1 pk > 0. To prove this
we take ai = 1 + bi and let q0 , q1 , . . . , qn be the elementary symmetric functions in
the bi s. To calculate the pk s in terms of the qj s, note that
n
Y

n
Y

(x + ai ) =

i=1

(x + 1 + bi )

i=1

gives
n
X

pk xnk

n
X

qj (x + 1)nj

j=0

k=0

n
X

qj

j=0

nj
X
k=0



n
n 
X
X
nj k
n j nk
x =
qj
x
;
k
nk
j=0
k=j

therefore, [exchanging the order of summation and equating coefficients]


pk =


k 
X
nj
j=0

nk

Thus, the difference pk+1 pk equals


rj

=
=

qj =


k 
X
nj
j=0

Pk+1
j=0

kj

qj .

rj qj , where

 




nj
nj
nj
nk
1

=
k+1j
kj
kj
k+1j


n j n 2k + j 1
kj+1
kj

(6)

for j < k + 1, and rk+1 = 1. Since ai is a positive integer, it follows that bi 0


and therefore, qj 0 and q0 = 1 > 0. If m n < 2m 3, then 0 < n m + 1 <
m 2 < m 1; therefore, we can set k = n m + 1, and n m < k < m 1 will
hold. Moreover 2k + 1 = 2(n m + 1) + 1 < n, so that, from (6) all rj are positive
[in particular, r0 > 0], and therefore, pk+1 pk r0 q0 > 0.For n = 2m3, putting
k = nm+1 = m2 again satisfies nm < k < m1. Also, 2k+1 = 2m3 = n,
so that, by (6) r0 = 0 but rj > 0 for j 1. Hence, pk+1 pk > 0 unlessq
 j = 0 for
all j 1; that is, bi = 0 for all i. But then all ai = 1, so that pk = nk for all k.

60

Editors note. Baron now computes the difference of the coefficients of m3


and and shows it is non-zero, which finishes the proof. But it is easier to
compare the coefficients of m2 and m3 : we get (since m 3 = n m) that
these coefficients are respectively pm2 and pm3 + p2m3 ; that is,



 



2m 3
2m 3
2m 3
2m 3
and
+
=
+1;
m2
m3
2m 3
m3

however it is known (and easy to see) that consecutive binomial coefficients nk
n
and k1
for n > 2 never differ by only 1, so that we are done.

Baron then proves part (a) separately (finding both solutions), since his
theorem does not apply to m = 3. He also states that if we allow some of the a i s
to be zero,there are two more solutions to (a), namely (0, 1, 1, 1) and (0, 0, 2, 2).

1362.

[1988 : 202]
Determine the sum
  
n X
n 
X
n
n n j2k

,
j
+
k
j
k
j=0
k=0

where w is a primitive cube root of unity.


Solution by G.P. Henderson, Campbellcroft, Ontario. [1989 : 249]
The coefficient of tn in
(1 + xt)a (1 + yt)b (1 + zt)c
is


X a b 
c
xi y k z njk .
j
k
njk

j+kn

Setting a = b = c = n, x = 1 , y = 2 , z = 1, we see that the required sum is


the coefficient of tn in



n
t
t
(1 + t) 1 +
1+ 2
= [(1 + t)( + t)( 2 + t)]n

= (1 + t2 )n .

Therefore, the sum is

n
n/3

or 0 according as n is or is not divisible by 3.

ATOM
A Taste Of Mathematics / Aime-T-On les Mathematiques
1. Edward J. Barbeau
(1995-1996 )
2. Bruce L.R. Shawyer

Mathematical Olympiads Correspondence Program


Algebra Intermediate Methods

3. Peter I. Booth, John Grant McLoughlin, and Bruce L.R. Shawyer


for Mathematics Leagues
4. Edward J. Barbeau, and Bruce L.R. Shawyer
5. Richard Hoshino, and John Grant McLoughlin

Problems

Inequalities
Combinatorial Explorations

6. Peter I. Booth, John Grant McLoughlin, and Bruce L.R. Shawyer


for Mathematics Leagues II

Problems

7. Andy Liu and Bruce L.R. Shawyer (Editors)


Canadian Collection Part 1

The Murray Klamkin Problems

8. Andy Liu and Bruce L.R. Shawyer (Editors)


Canadian Collection Part 2

The Murray Klamkin Problems

9. Andy Liu and Bruce L.R. Shawyer (Editors)


Canadian Collection Part 3

The Murray Klamkin Problems

10. Andy Liu and Bruce L.R. Shawyer (Editors) The Murray Klamkin Problems
Canadian Collection Part 4
Cost per volume (including shipping and handling charges):
Regular Rate $12.00 CMS Members Rate $9.00 (excluding taxes).
For non-Canadian shipping addresses the rates quoted must be paid in US funds.
Co
ut par volume (frais y compris dexpedition et de manutention) :
Taux regulier 12,00 $ Tarif dmembre de la SMC 9,00 $ (avant taxes).
Pour une adresse a
` letranger, paiement requis en devises americaines.
For more information and to order:
Pour plus dinformations et pour completer une commande :
www.cms.math.ca/Publications/books
or contact:

ou contacter :
CMS/SMC Publications
577 King Edward, Ottawa (Ontario) Canada K1N 6N5
Email: publications@cms.math.ca
Courriel : publications@smc.math.ca

Canadian Mathematical Society / Societe mathematique du Canada


577 King Edward, POB/CP 450-A
Ottawa, Ontario, Canada K1N 6N5
www.cms.math.ca

Vous aimerez peut-être aussi